Divisional of a Divisional: Avoid Terminal Disclaimers

Boehringer Ingelheim International GMBH v. Barr Laboratories, Inc. (Fed. Cir. 2010) (denial of en banc rehearing)

In January 2010, the Federal Circuit expanded the scope of the safe-harbor created by 35 USC 121 against double patenting allegations. Namely, the court held that a divisional-of-a-divisional still qualifies for the safe harbor so long as the later-filed applications do not violate the Examiner’s original grouping suggestions.

Now, the court has denied generic drugmaker Mylan request for rehearing en banc. Mylan had presented two questions:

1.    Whether a divisional of a divisional patent application is a “result of a restriction requirement imposed on an original application, entitling a patentee to invoke 35 U.S.C. § 121’s safe harbor from invalidity due to double patenting, where the second divisional was filed voluntarily for strategic reasons and was not mandated by the original restriction requirement.”

2.    Whether a divisional application is “consonant” with a restriction requirement, entitling a patentee to invoke the safe harbor of 35 U.S.C. § 121, where the divisional disregards the lines drawn by the examiner in imposing the restriction requirement.

Judges Gajarsa and Dyk dissented from the en banc denial. In the dissent, Judge Gajarsa wrote:

Because the majority’s decision improperly expands the statutory safe-harbor provision of 35 U.S.C. § 121 beyond Congress’s intended scope, I respectfully dissent from the court’s denial of Mylan’s petition for rehearing en banc. While the Supreme Court has not construed § 121, the majority’s expansive opinion is inconsistent with our longstanding precedent, and will work a major change in our jurisprudence.

The purpose of § 121 is to prevent the inequity that resulted from an examiner’s improper restriction requirement (separating out supposedly patentably distinct inventions that were in fact the same or not patentably distinct), whereby a patentee’s compliance with an examiner’s incorrect restriction would result in the original application being used as a reference against the later divisional application and a rejection on double patenting grounds. See Pfizer, Inc. v. Teva Pharms. USA Inc., 518 F.3d 1353, 1361 (Fed. Cir. 2008); Studiengesellschaft Kohle mbH v. N. Petrochem. Co., 784 F.2d 351, 359 (Fed. Cir. 1986) (Newman, J., concurring). Thus, § 121 “effects a form of estoppel” against the PTO, “that shields the applicant from having to prove the correctness of the restriction requirement in order to preserve the validity of the second patent.” Studiengesellschaft, 784 F.2d at 361. In our past decisions, we have limited § 121 in accordance with its language and clear purpose to situations in which the patent applicant has been forced to divide his application by action of the PTO. We have interpreted the statute to include a requirement of consonance, requiring that the later application or applications follow the original examiner’s restriction requirement, and we have interpreted the statute to include a requirement that the later application or application be filed “as a result of” the original PTO restriction.

In addressing § 121, the majority opinion commits two legal errors. First, it significantly undermines this court’s “consonance” precedence by permitting a patentee to ignore the examiner’s demarcation of independent and distinct inventions in subsequent divisional applications. Second, it engages in an impermissible and expansive reading of § 121’s “as a result of” language. Either error in isolation would be worthy of en banc review; coupled together, the errors threaten to significantly broaden § 121’s safe-harbor provision in clear defiance of this court’s “strict test for application of § 121.” Geneva Pharms., Inc. v. GlaxoSmithKline PLC, 349 F.3d 1373, 1382 (Fed. Cir. 2003).

 

156 thoughts on “Divisional of a Divisional: Avoid Terminal Disclaimers

  1. 156

    Ned,

    I have thoroughly trashed the case law you have presented. Unless you got more, you sound like Gajarsa – lame.

    121 simply is not the monster that needs even near the cabining it did back in the submarine times. It is clearly meant to protect agains Office screw-ups, of which were plenty in this case – don’t forget that “but for” the Office violation of its guarantees amounting to four and a quarter years (which I haven’t seen any Office apologists care to defend or explain), the immediate case would be moot.

    I would say your vastly overstate your version of the primary need. The majority simply got it right.

    Let go.

  2. 154

    Well, 6, the courts, until the present case, uniformly held that Section 121 was to be narrowly construed as an exception to double patenting. The reason for this narrow construction was to strictly cabin the use of section 121 to prevent the unjustified extension of the patent monopoly in time. These concerns, once of great concern given that patents extended in time 17 years from issuance, are of less concern today given that patents expire 20 years from their US effective filing dates. But, given that terms can be extended up to 10 years, and the fact that without a terminal disclaimer, patents on the same invention could be owned by two different enities, the remaining concerns are not deminimis.

    So the primary need remains to prevent double patenting. It is the public who needs to have Section 121 narrowly cabined. The current ruling seems to have thrown all prior case law on this issue into the trashcan by eviserating both the requirement of strict consonance and the statutory requirement that a divisional had to be filed as a result of the restriction.

  3. 153

    Ned,

    I will also offer the following observations:

    Who is meant to be protected by 121?

    From whom is the protection of 121 meant to be?

    What is the originating action that is meant to initiate 121 protection?

    Do you catch the drift that 6 misses from my hypo?

    You also might go back and read the blistering dissent in your own case cite of Bristol-Meyers. Think of what that judge may have had to say on the immediate case’s fact pattern (hint: the Office wouldn’t be sitting for a week).

  4. 152

    I am uncertain of your position

    So?

    I asked pong to clarify and he did.

    Goody for Pong. Goody for you.

    How would you answer?

    I done tol’ you – Homey don’t do answers.

    Although I will offer the observation as I did at May 12, 2010 at 11:00 AM above concerning “implicit” and exclusive action by the Office concerning any alleged understanding in relation to which there is disagreement or doubt. What do you think that means?

    6,

    Because we don’t restrict specs tard.

    And obviously you don’t write them either. And just as obviously you miss the drift on the whole B-C separation and consonance argument. Yous need to go back and pay attention to IANAE – he knows consonance.

    Further, in essence you have just agreed that the actual case at hand was rightly decided. Talk about wasting your time Tard – try to actually think through your logic before making a stand.

  5. 151

    ping, I am uncertain of your position. I asked pong to clarify and he did. How would you answer?

    Pong quoting Ned “Now, what I do not know is your position on B vs. C in a case where B and C are filed in the same divisional, are not restricted, but are examined together and perhaps even allowed, and only thereafter, instead allowing granted C to issue, the applicant voluntarily, without requirement by the examiner, cancels claims to C and presents them in a third application. When the patent to B issues, can the examiner enter a ODP rejection of C vs. B?”

    Pong replying to Ned: “Ned, my position is: no. B and C are independent or distinct as set forth in the restriction requirement. The fact that the examiner of the divisional containing B and C did not restrict them is meaningless. The failure to restrict is not an “implicit withdrawal” of the determination that A and B and C are all independent or distinct from one another. Only ftards like 6 think that. But what do you expect from somebody who can’t even grasp a simple concept like consonance?

    Posted by: pong | May 12, 2010 at 01:57 PM”

  6. 150

    “If someone wanted to follow the letter and spirit of a divisional request, why wouldn’t the spec and claims be equally separated?”

    Because we don’t restrict specs tard.

    I can’t believe I even wasted my time talking to you. I should have known from the outset that you had no point to make what so ever that was relevant to the issues here.

  7. 149

    Sure, if you leave out C in the spec of B then you’re fed. Anyone who is re tarded enough to do that deserves it.

    But 6 – point of the hypo was to show that your R-tarded “B must be separate from C” hypothosis was R-tarded. If someone wanted to follow the letter and spirit of a divisional request, why wouldn’t the spec and claims be equally separated? Or do you always recommend writing specifications to more than one distinct invention? You be all high and mighty about the applicant not f’ing up and trying to claim B and C in the second app cause that don’t follow your thinkin on consonance, yet you more than fine with lacking consonance in the spec – what happened to your spirit, bro?

  8. 148

    Well I have B
    And also C
    And two attests,
    I clearly agree.
    it’s all there in the history.
    WHAT UP WITH THAT?

  9. 147

    ” that means that C is not claimed and is not in the spec of the second application.”

    AND IT IS NOT IN THE SPEC OF THE SECOND APP?

    Why the f would it not be in the second apps spec? What kind of re tarded hypo are you trying to craft here? What is the point of it?

    Sure, if you leave out C in the spec of B then you’re fed. Anyone who is re tarded enough to do that deserves it.

  10. 145

    This ha_te filter gotsa go.

    6,

    I have pinpointed your error for you. You have conflated the real case with my hypo and you made a critically wrong ASS-U-mption.

    Your statement of “ I already said the chain goes like this: File app 1. Get restriction to A B C. Pick A no traverse. File app 2 (implicitly with the same SPEC as app 1 you tardface) and claims only to B. is not the question I asked you. Pay attention sunshine.

    So while you may have given an answer more than once, it was an answer to the wrong question.

    When I state “that second application does not contain both inventions B and C.” that means that C is not claimed and is not in the spec of the second application. Your bridge is out of order.

  11. 144

    B Call me.
    Maybe it’s late but just call me.
    Don’t be afraid,you can call me.
    Call me and we will sit down.

    If I’m what you were protecting,
    maybe now I am detecting,
    that you really were selecting
    to help MEEEEEEEEEEEEEE.

    Call me.
    maybe it’s late, but just call me,
    Don’t be afarid. You can call me.
    Call me and we can sit down.

  12. 143

    You know who else didn’t like restriction requirements?

    Posted by: Malcolm Mooney | May 12, 2010 at 01:44 PM

    I was going to guess Mike Godwin, but he doesn’t seem to have any patents.

    Posted by: IANAE | May 12, 2010 at 01:57 PM

    The correct answer is Hit-ler.

  13. 142

    Ping’s idea of a “challenge” is: Ask someone to do something and then when they do it simply state that they didn’t without explanation or reason, and state that the challenge is still going!

    You also have to say “since it’s your own fault you haven’t done what I asked, this challenge is made final”.

  14. 141

    “And 6 – your handwaiving of 120 and 121 don’t meet the challenge. ”

    What challenge you tard? Your “question” about “how” has already been answered, in DETAIL by the recitation of an entire procedure. Twice. Simply state what is the matter with my procedure in terms of what specific provision of law would prevent it from happening. It should take you less than a paragraph.

    The only “challenge” remaining is for you to tell me what is wrong with my answer. Specifically. In terms of what would be wrong with the 3rd app and what would happen to it because of a specific provision in the law.

    Ping’s idea of a “challenge” is: Ask someone to do something and then when they do it simply state that they didn’t without explanation or reason, and state that the challenge is still going!

  15. 140

    So, are we in agreement?

    We are in agreement that IANAE is the man! Nobody and I mean nobody spins a better response with absolutely no substance better than he.

    Notwithstanding that, pong should take that volley and smash it like the worhtless crap non-answer, non-point gibberish that it is.

    I see you’ve elected the set of strawman claims for further prosecution on the merits.

    Acceptance at this point isn’t where the fun starts – it starts in the next app and what is included in the spec and claims – different ballgame (as long as no claims are repeated to this first acceptance).

    And 6 – your handwaiving of 120 and 121 don’t meet the challenge. I hope this don’t turn into another Court adventure….

  16. 139

    pong,

    I see you’ve elected the set of strawman claims for further prosecution on the merits.

    I hereby withdraw all the other claims I made, because responding to actual arguments seems to be a burden for you. Of course, I’m not admitting that any of them cause you any burden at all, and I’m not saying I agree with you in any way. Please note how I’m conspicuously not disagreeing with you while acting exactly like a person who agrees with you, at exactly the best possible time for a person who disagrees with you to voice any concerns he may have.

    So, are we in agreement?

  17. 137

    “You’re not agreeing to that. You’re agreeing that there is a serious burden between any and all of them, not just 3 of them together.”

    Incorrect.

    “And, in the event that you think that is an issue, maybe I’ll have to get our FP writers to go ahead and specify that such is the case.”

    I won’t hold my breath waiting for the FP writers to fish your suggestion out of the circular file it will be immediately placed into.

  18. 136

    “Under the new IANAE/6tard legal theory of “implicit withdrawal”?

    The answer to that could not be more clear: NO.”

    Nobody even ever said they could. I know how you love straw men. From behind.

  19. 135

    “I agree that there would be a serious burden to search and examine all 3 in one application”

    You’re not agreeing to that. You’re agreeing that there is a serious burden between any and all of them, not just 3 of them together. And, in the event that you think that is an issue, maybe I’ll have to get our FP writers to go ahead and specify that such is the case.

  20. 134

    “What is really happening is that the applicant is no longer being required to restrict in the way that he was previously. The PTO is letting him present two sets of claims in one application, but he is choosing to separate them into two separate applications. How can that action by the applicant logically be characterized as the result of any action by the PTO?”

    What’s your point?

    The issue is: using 6tard’s original example, if the applicant elects A without traverse, and then files ONE divisional application containing both B and C, can the examiner of the divisional application (regardless of whether it’s the same examiner as the parent or a different examiner) not restrict B and C and then reject the divisional application under ODP over the parent? Under the new IANAE/6tard legal theory of “implicit withdrawal”?

    The answer to that could not be more clear: NO.

  21. 133

    “Only ftards like 6 think that. But what do you expect from somebody who can’t even grasp a simple concept like consonance?”

    I have already told you that I don’t think that it was implicitly withdrawn tardface. And apparently I understand consonance a fton better than you do.

    In you two’s little hypo here there sure as f is no consonance in app 2 if the examiner fails to restrict. They are independent and distinct, as you noted, from the first app. And also, burden was previously established.

    In fact, let me quote you the old form paragraph. The new one isn’t much different but I don’t have a copy off hand.

    “Restriction for examination purposes as indicated is proper because ALL THESE inventions listed in this action are independent or distinct for the reasons given above AND there would be a serious search and examination burden if restriction were not required because one or more of the following reasons apply:
    (a) the inventions have acquired a separate status in the art in view of their different classification;
    (b) the inventions have acquired a separate status in the art due to their recognized divergent subject matter;
    (c) the inventions require a different field of search (for example, searching different classes/subclasses or electronic resources, or employing different search queries);
    (d) the prior art applicable to one invention would not likely be applicable to another invention;
    (e) the inventions are likely to raise different non-prior art issues under 35 U.S.C. 101 and/or 35 U.S.C. 112, first paragraph. ”

    Note the “all these” and “and” that I bolded.

  22. 132

    “If the examiner is saying that the applicant must elect only one of A, B, C, he is saying that any two of those three would pose a serious burden if examined together.”

    So? What if the examiner is incorrect about the burden of searching B and C together? If I’m electing A without traverse because I agree they are independent or distinct as claimed and I agree that there would be a serious burden to search and examine all 3 in one application, I’m not required to “traverse” the restriction between B and C. Only a f’in tard like 6 thinks that.

    “If the examiner later allows B and C to be presented in the same child application without restricting, he is saying (one would think (if one didn’t sit on the CAFC bench)) that from that point forward no further divisional separating B and C is a result of any previous requirement for restriction, because the applicant no longer has any obstacle to presenting B and C together.”

    So my first divisional application containing B and C issues with claims to B and C. I then file a third application (second divisional) with claims directed to B’. Yes, the third application (second divisional) can possibly be rejected under ODP over the second application (first divisional), but not over the first application (original application).

  23. 131

    The failure to restrict is not an “implicit withdrawal” of the determination that A and B and C are all independent or distinct from one another.

    That’s just one way you might try to understand it. It’s not what anybody claims is really happening.

    What is really happening is that the applicant is no longer being required to restrict in the way that he was previously. The PTO is letting him present two sets of claims in one application, but he is choosing to separate them into two separate applications. How can that action by the applicant logically be characterized as the result of any action by the PTO?

  24. 130

    “Now, what I do not know is your position on B vs. C in a case where B and C are filed in the same divisional, are not restricted, but are examined together and perhaps even allowed, and only thereafter, instead allowing granted C to issue, the applicant voluntarily, without requirement by the examiner, cancels claims to C and presents them in a third application. When the patent to B issues, can the examiner enter a ODP rejection of C vs. B?”

    Ned, my position is: no. B and C are independent or distinct as set forth in the restriction requirement. The fact that the examiner of the divisional containing B and C did not restrict them is meaningless. The failure to restrict is not an “implicit withdrawal” of the determination that A and B and C are all independent or distinct from one another. Only ftards like 6 think that. But what do you expect from somebody who can’t even grasp a simple concept like consonance?

  25. 127

    pong, just for the record, I agree with you regarding the safe harbor of both B and C vs. A, even if B and C are filed in the same divisional.

    Now, what I do not know is your position on B vs. C in a case where B and C are filed in the same divisional, are not restricted, but are examined together and perhaps even allowed, and only thereafter, instead allowing granted C to issue, the applicant voluntarily, without requirement by the examiner, cancels claims to C and presents them in a third application. When the patent to B issues, can the examiner enter a ODP rejection of C vs. B?

  26. 126

    “No he didn’t you f’in tard. He established that there would be a serious burden to search and examine an application having claims to A, B, and C.”

    Hardly. That is the stupidest fin interpretation of a restriction I’ve ever heard. The BURDEN must be established with respect to each group in relation to every other group in order for the restriction to be proper.

    “As ping already explained to you, the PTO’s actions are based exclusively on the written record.”

    The what causes a PTO action has nothing to do with whether or not the restriction is implicitly lifted. God you’re stup id as fu ck.

    “There is no such thing as an “implicit finding” you tard.”

    Fine with me, I hope they don’t make that implicit finding. Although dollars to pennies I bet they do. Whether or not you say they will or not.

    “If the parent application contains a WRITTEN restriction requirement determining that A and B and C are independent or distinct from one another, than absent a WRITTEN retraction of that determination, A and B and C are independent or distinct from each other for eternity.”

    So, you believe they’re independent and distinct, but since they were represented in app 2 as B and C, then they’re entitled to the examiner reevaluating burden between those two groups EVEN THOUGH HE ALREADY DID SO IN APP 1 in order for the restriction to even be proper. You sir are a re tard. What’s more, as IANAE points out, the “burden” nonsense is policy AT BEST, it has nothing to do with what the law requires or grants (ahem ODP protection). It is 100% irrelevant to the issues in the instant case.

    “And the examiner can’t, in the divisional application, withdraw or retract, either “implicitly” or in writing, the restriction requirement made in the parent in order to reject the divisional over the parent using ODP. That’s exactly why section 121 was enacted.

    Fine with me, doesn’t support your point about the instant app, or ping’s hypo in the least bit. It in fact supports mine.

    Apparently you’re re tarded enough to believe that ping’s position with regard to the case at hand is correct based on nothing but the supposed fact that the examiner didn’t establish burden between B and C in app 1. You’re out of your mo fin gourd son.

  27. 125

    He established that there would be a serious burden to search and examine an application having claims to A, B, and C. He didn’t establish squat about a serious burden for an application containing B and C.

    If the examiner is saying that the applicant must elect only one of A, B, C, he is saying that any two of those three would pose a serious burden if examined together.

    If the examiner later allows B and C to be presented in the same child application without restricting, he is saying (one would think (if one didn’t sit on the CAFC bench)) that from that point forward no further divisional separating B and C is a result of any previous requirement for restriction, because the applicant no longer has any obstacle to presenting B and C together.

  28. 124

    Ping in response to your continual annoying posts I’m going to go ahead and breifly reply.

    “really? – show me the lawl lol on this sunshine – especially given an application 2 that only has B.”

    Applicant 2 has the same spec as app 1. It has A B C in the SPEC. It has B in the CLAIMS.

    Here is the law as to how you can chain back applications.

    link to uspto.gov

    link to uspto.gov

    If there is something specifically in these laws that you feel like would preclude filing in the fashion I’ve laid down then go ahead and spit it out lazy arse.

    “Show me how a patent that cannot have children can now have a divisional through an active child when that child is absent the substance of the grandchild (this be like a virgin birth).”

    It doesn’t “have a DIV” quote unquote “through an active child”. The grandchild DIV is a DIV of the child, where the restriction is still in force by the way the wording of 121 is stated.

    “If the other invention is made the subject of a divisional application which complies with the requirements of section 120 of this title it shall be entitled to the benefit of the filing date of the original application.”

    Do you see this? This means that if you file an appliction, for instance application 3, and call it a DIV, you may also put in a claim to the priority of app 2 and app 1.

    “A patent issuing on an application with respect to which a requirement for restriction under this section has been made, or on an application filed as a result of such a requirement, shall not be used as a reference either in the Patent and Trademark Office or in the courts against a divisional application or against the original application or any patent issued on either of them, if the divisional application is filed before the issuance of the patent on the other application.”

    Do you see this? This means that the original app cannot be used against app 3 in an ODP rejection.

    Now, what, specifically, do you think prohibits someone from filing in the manner I laid out for you?

    ” that’s just it 6 – this can only happen if what you want in application 3 (invention C) is in application 2 (inventions B and C). No way no how can you depend on application 2 if that application does not contain B and C. Any use of application 2 that only has B cannot give this mythical bridge that you think it can. Now the question you still haven’t answered is how you do this – don’t give me no empty words that Iza can – tell me exactly how.”

    App 2 has the same spec as app 1 and in that regard has C in it. App 2 however does not have C in the CLAIMS. Why you think this is some big ol’ hairy deal as to this being something that would prevent app 3 from getting the priority date and the ODP protection is strictly up to you to explain.

    As to “how” I do this, I just laid out for you the procedure, what specifically do you think goes wrong at the 2nd stage?

    “in case you missed it 6 – oh wait – you did.”

    I’m pretty sure I didn’t miss jack sht you tard. I already said the chain goes like this:

    File app 1. Get restriction to A B C. Pick A no traverse. File app 2 (implicitly with the same SPEC as app 1 you tardface) and claims only to B. Let app 1 issue. File app 3 with claims to C, claim priority to app 1 by claiming priority to app 2 and 1, and call it a DIV, you will get ODP protection.

    That is the HOW you do this. That is the procedure. I’ve seen it happen tardface. And yes I will cite you some apps that do this. It won’t be like a fin virgin birth at all. It’ll be more like the 1000000 results I’ll get by searching “(which NEAR divisional) AND (which near divisional) AND (which NEAR divisional)” in EAST.

  29. 123

    “He already ‘established’ that in the first app. Ya tard. You didn’t traverse.”

    No he didn’t you f’in tard. He established that there would be a serious burden to search and examine an application having claims to A, B, and C. He didn’t establish squat about a serious burden for an application containing B and C.

    “No reason, it’s a complete bullsht implicit finding like so many of the things the judiciary does, or would very likely do, to make up for incompetence on the part of applicants.”

    There is no such thing as an “implicit finding” you tard. As ping already explained to you, the PTO’s actions are based exclusively on the written record. If the parent application contains a WRITTEN restriction requirement determining that A and B and C are independent or distinct from one another, than absent a WRITTEN retraction of that determination, A and B and C are independent or distinct from each other for eternity.

    And the examiner can’t, in the divisional application, withdraw or retract, either “implicitly” or in writing, the restriction requirement made in the parent in order to reject the divisional over the parent using ODP. That’s exactly why section 121 was enacted.

    Damm you iz dumm.

  30. 122

    “Why can’t it simply “say” that the examiner can’t establish that a serious burden does not exist to search and examine both B and C in the same application?

    He already “established” that in the first app. Ya tard. You didn’t traverse.

    “Why would the examiner’s decision to not restrict B and C in a divisional “unsay” that? ”

    No reason, it’s a complete bullsht implicit finding like so many of the things the judiciary does, or would very likely do, to make up for incompetence on the part of applicants.

  31. 121

    “What seems clear is that one strongly held view has been shown to be wrong, but is still being clung to.”

    I don’t know where you think you showed jack to be “wrong”. You ain’t showed sht son.

    Apparently my post from last night did not post so I will retype it later.

  32. 120

    What is clear is that there seem to be two views on this topic, strongly held.

    What seems clear is that one strongly held view has been shown to be wrong, but is still being clung to. The call for en banc is clearly not needed, because the cases cited are distinguishable. Have you scheduled yourself for a class with the PTO disciplinary authorities yet?

    As to anything “implicit”, the Office is forstalled from that avenue by some silly rule. Something about being clear on the record…

    Oh yeah, “The action of the Patent and Trademark Office will be based exclusively on the written record in the Office. No attention will be paid to any alleged oral promise, stipulation, or understanding in relation to which there is disagreement or doubt.

    Shamelessly stolen, but at least I don’t need 4 pages to expound on it.

  33. 119

    What is clear is that there seem to be two views on this topic, strongly held. The Feds should have taken this case en banc. But since they did not, the PTO needs to help, by rule.

    I would suggest this:

    1) The office will consider a restriction requirement to carry over to a granddaughter if and only if strict consonance is maintained in intervening applications.

    2) If strict consonance is not maintained, the restriction requirement will carry over to a granddaughter application if and only if the restriction requirement is specifically referenced by the examiner in the daughter application and there reimposed.

    3) Define consonance to mean strict and exclusive compliance with the examiner’s demarcation of groups. Claims in a non elected group will have to be set forth verbatim. New claims may be added, but such new claims are not presumed to be consonant.

  34. 118

    “So pretend I said that.”

    I could if I would. But you didn’t say that.

    “In any case, it’s not important to my argument which of the conditions for restriction is implicitly absent when the PTO chooses not to restrict.”

    It’s not? So, using 6tard’s example, if you elected A in the parent without traverse, and filed a divisional including both B and C and the examiner didn’t restrict, but gave you an ODP rejection over the parent, on the theory that he/she has “implicitly withdrawn” the restriction requirement (i.e. has “implicitly” determined that the original determination that A, B, and C were all independent or distinct is now no longer true), you’re telling me it’s not important why the examiner did not restrict B and C in the divisional?

    This entire “implicit withdrawal” exchange between you and 6tard is really annoying. There is no such thing. If you had simply told him that in your reply then maybe the rest of us wouldn’t have to pretend that you know what you’re talking about.

  35. 117

    There are TWO requirements for a proper restriction: 1) the inventions as claimed are independent or distinct AND 2) there would be a serious burden on the examiner if restriction was not required.

    I’m pretty sure I’ve had this conversation once before.

    Two inventions can properly be restricted under the statute for the sole reason that they are patentably distinct. The undue burden thing is only in the MPEP. Not that I mind it as a requirement, but I consider it a mere statement of policy that could be changed or ignored at any time. The actual law says that if you have two inventions you should be filing two applications.

    Also, I’ve never had an examiner show me much in the way of undue burden. Usually just a conclusory statement or a (sometimes very general) suggestion that the claims would be in different art classes.

    In any case, it’s not important to my argument which of the conditions for restriction is implicitly absent when the PTO chooses not to restrict. The important thing is that the PTO no longer requires restriction between B and C. So pretend I said that.

  36. 115

    “In your scenario, you wouldn’t be allowed to serial chain divisionals and applicants would haveta claim all divisionals offa Application 1 before Application 1 issues.

    Serious lack o’ understandig the law there 6.”

    Truer words were never spoken.

  37. 114

    “the other INVENTION (SINGULAR) is made the subject of a divisional application which complies with the requirements of section 120 of this title it shall be entitled to the benefit of the filing date of the original application.

    The other invention (singular) is taken care of in a DIV one at time under 121 bucko.”

    LOL

    Your ability to “interpret” statute gets even more hilarious with time.

  38. 113

    “That failure to restrict ‘un-says’ that B is a different invention from C.”

    Why?

    Why can’t it simply “say” that the examiner can’t establish that a serious burden does not exist to search and examine both B and C in the same application?

    The examiner “found” in the parent that A was distinct from B and C, that B was distinct from A and C, and that C was distinct from A and B, and presumably presented reasons for those “findings.” Why would the examiner’s decision to not restrict B and C in a divisional “unsay” that?

    There are TWO requirements for a proper restriction: 1) the inventions as claimed are independent or distinct AND 2) there would be a serious burden on the examiner if restriction was not required.

    Regardless of whether the examiner of the divisional is the same as or different from the examiner of the parent who wrote the restriction requirement, the “official position” of the PTO is that A is independent or distinct from B and C, B is independent or distinct from A and C, etc.

    So how does not restricting B and C in the divisional “unsay” that B and C are independent or distinct?

    Please explain.

  39. 112

    Application 1 is allowed and prior to issue, applicant files a divisional application 2

    in case you missed it 6 – oh wait – you did.

    Naw, because of the way the filing date provisions in the law work out you’ll be fine.

    Is this addressing any point I made?

    stating that app A is closed from having any children is equivalent to the statement that no app could claim priority back to it. That is not the case.

    really? – show me the lawl lol on this sunshine – especially given an application 2 that only has B. Show me how a patent that cannot have children can now have a divisional through an active child when that child is absent the substance of the grandchild (this be like a virgin birth).

    App 2 can build a bridge for the priority to flow” – that’s just it 6 – this can only happen if what you want in application 3 (invention C) is in application 2 (inventions B and C). No way no how can you depend on application 2 if that application does not contain B and C. Any use of application 2 that only has B cannot give this mythical bridge that you think it can. Now the question you still haven’t answered is how you do this – don’t give me no empty words that Iza can – tell me exactly how.

    Ned,

    Don’t grant me anything – it’s the law you offended – She don’t like being mistreated, and mis-citing be mistreating (just a tad more than your crappy I purposely avoided that type of language argument that was, shall we say, self-evidently spurious.

  40. 111

    ping, I checked 07/124,197 on PAIR. It does list a 1987 a preliminary amendment filed on 11/23/1987, the filing date of US’197, which seems to suggest they may have cancelled the claims elected in the parent on filing the divisional.

    So, I will grant you this. The ‘197 was a divisional to the extent that filing claims not in consonance with the restriction requirement was proper.

  41. 110

    I just wanted to stop by and say:

    link to youtube.com

    “NOW, how do you get both inventions B and C when Application 1 is closed to further children?”

    Um, you don’t? If app 1 issued without you filing your app 2 or 3 then as far as I know you’re fed. Hopefully you at least filed app 2.

    “In your scenario, you wouldn’t be allowed to serial chain divisionals and applicants would haveta claim all divisionals offa Application 1 before Application 1 issues.”

    Naw, because of the way the filing date provisions in the law work out you’ll be fine.

    “So, to restate my question, tell me how an applicant can obtain a third patent for invention C from the second application, if that second application does not contain both inventions B and C. Application 1 is no longer allowed to have children.”

    Here’s the deal pingersloth, you can file whatever claims in whatever application whenever you want, and you can claim priority back to whatever the f you want. Whether or not you actually get your priority date and whether or not you get your 121 ODP protection depends on whether or not you meet certain req’s. Those reqs happen to be set out in the statute.

    In your situation you’re trying to make the false argument that stating that app A is closed from having any children is equivalent to the statement that no app could claim priority back to it. That is not the case. App 2 can build a bridge for the priority to flow. And the restriction req can build the proper bridge to put C in app 3.

    In your scenario you file app 3 after app 1 issued and make your claim back for priority, and you happen to get it because the law allows you to. You will also happen to get ODP protection because the law also allows that. If you disobey the laws on claiming back for priority and/or the laws on getting ODP protection then you do not recieve either or.

    In the instant case at issue, it appears that the applicant broke the laws on getting ODP protection.

    “The LAWL saw the problem and said, ya’know what – the Office is goin to screw the applicant and weza gotta protect ‘im. Thus 121 was born.”

    Then they did a piss poor job o it then.

    And ffs, note that the office didn’t screw anyone in any of these situations, your hypo or the instant case. In the instant case the applicant decided to blunder along like a big ol’ boob doing whatever he pleased rather than following the law to get the benefits he wanted. He could have perfectly easily gotten the benefits he wanted legitimately by filing DIVs properly. JUST LIKE THOUSANDS OF OTHER PEOPLE DO EVERY FRAKIN DAY I SHOULD ADD.

    Be clear ping, you people arguing for this nonsensical reading of the law might have gotten a panel majority (we’ll see come friday when the decision issues) but you are by far in the minority so far as I can see from the way DIVs are filed in my cases. And I will not be surprised to see another case like this come along here shortly to straighten this sht right straight out.

    Do you think that all those other people are less sophisticated than you? Do you think they simply want to file more apps at a substantial cost to themselves? I can only imagine we’re talking 100’s of thousands o dollars for individual apps. Why do you reckon most folks don’t want to try this risky business? They just want to string prosecution along longer? Could be, but from the numbers involved I’m doubting it.

  42. 109

    6 the typo I have is that I was just plain lazy to get all cute with “app 1, app 2 app 3, invention A, invention B, invention C, app A, app B, app C, invention “A”, invention “B”, invention “C” – I just used A, B, and C for both app and invention, thinking that maybe you were a smart fella and could figure that out. I guess I was wrong about you being a smart fella.

    You also do not answer the first question which prompted this hypo discussion which was about serial claiming. So let me get unlazy for a moment and put the question in language even you might understand.

    Application 1 receives a restriction to invention A, invention B and invention C.
    Applicant responds by choosing invention A for Application 1.
    Application 1 is allowed and prior to issue, applicant files a divisional application 2 (we’ll hold off a moment before looking at what is filed in application 2).
    Application 1 issues.

    NOW, how do you get both inventions B and C when Application 1 is closed to further children?

    As pong has bounced the ball off your noggin (nice game pong), you seem all bent out of shape that Application 2 may have been (and in the analogous case, was) filed with both inventions B and C. As my main man IANAE esplained to you – consonance is only required for the separation of the allowed patent and “everything else”, so that the first allowed patent don’t have no second patent coming up and claiming whats in the first allowed patent (it don’t say nothin about nothin else – as you only be too happy to point out – other examiners may feel different than the first).

    In your scenario, you wouldn’t be allowed to serial chain divisionals and applicants would haveta claim all divisionals offa Application 1 before Application 1 issues.

    Serious lack o’ understandig the law there 6.

    So, to restate my question, tell me how an applicant can obtain a third patent for invention C from the second application, if that second application does not contain both inventions B and C. Application 1 is no longer allowed to have children.

    I can’t say that I know what the correct course of action for an examiner to take here is

    No duh.

    You also says “ the later option that can get tricky because if that is assumed to be true then there might not be a basis for having restricted A from B and/or C in app 1 and if you withdraw that restriction as well, well idk, sht hits the fan.

    The LAWL saw the problem and said, ya’know what – the Office is goin to screw the applicant and weza gotta protect ‘im. Thus 121 was born.

    Pong,

    Not really an answer.

    You expected a real answer from 6? I lol you.

    I thought you said you understood restriction.

    Why would you understand that?

  43. 108

    Bottom line, your reliance on the implicit withdrawal is misplaced in a complicated restriction where the applicant simply went ho g wild with how he filed DIV’s and where there are way too many consequences to allow for the “oops” factor that you’re really relying on to make that implicit finding of withdrawal of the original restriction. If the examiner isn’t saying “oops” and is instead saying “w t f I need to get a case out”, then you shouldn’t be getting 121 ODP protection anyway.

    And again, if you make that implicit withdrawal of restriction at the issuance of an action in the instant 2nd app, what makes us believe then that the original restriction should not have been done away with in total? What then would shield and provide for the filing of your new app 3?

  44. 107

    Bottom line, your reliance on the implicit withdrawal is misplaced in a complicated restriction where the applicant simply went ho g wild with how he filed DIV’s and where there are way too many consequences to allow for the “oops” factor that you’re really relying on to make that implicit finding of withdrawal of the original restriction. If the examiner isn’t saying “oops” and is instead saying “w t f I need to get a case out”, then you shouldn’t be getting 121 ODP protection anyway.

    And again, if you make that implicit withdrawal of restriction at the issuance of an action in the instant 2nd app, what makes us believe then that the original restriction should not have been done away with in total? What then would shield and provide for the filing of your new app 3?

    Notice also that the fact of the matter is that you’re just trying to take advantage of any st upid examiners that you can find. If they wanted to withdraw the restriction of your claims then that shouldn’t be an implicit finding by way of them (perhaps accidentally) examining all claims, it should be a finding only if they state that they did such in their action.

    BTW, in the instant case it has already apparently been fairly well established that the app didn’t do some simple, oh, well I’ll put a few claims in and see if he won’t restrict. He specifically tried to avoid an interference with his little manuevering. I see even less reason to make that implicit finding as you would have us make. We passed “no reason to make that implicit finding” a long time ago.

  45. 106

    “We’ll take that as an implicit statement by the PTO that B and C aren’t so different after all.”

    As I have already stated, I have little issue with that little assumption in the simplistic little case where it is ABC. And I say that even though there is no basis in the law or any reason to assume that implicit statement. It could very well be that the examiner needed to act on the case even if there was a burden and they were independent and distinct. In which case there is no reason to believe that he wished to withdraw the restriction as to the remaining groups. Perhaps he should ODP them and be done with them lol. Just forget I suggested that last thing lol 😉

    Where, however, as in the instant case, you have applicant just jumbling sht all up however he feels like he wants to, he doesn’t get 121 protections from ODP as his claims are clearly not consonant at all, and what is more, you’d like to have me believe that the restriction was implicitly lifted before app 3 was even filed. He, by making up his own groups, clearly is not following the restriction, and he is not properly making a DIV. What “invention” according to the original restriction was app 3 in response to? None. He made up his own hypothetical grouping. He didn’t just present all claims and see what fell out like a re tard. He gets no implicit finding that he is a re tard and the examiner wanted to be his buddy (or however you want to state the flimsy implicit finding you’re relying on).

    And if we assume that the restriction is lifted after what he got in App 2 then App 3 shouldn’t even be able to be a DIV of app 2, it would have to be a CON of app 2 if anything because there is no restriction req anymore.

    There is thus even less reason to imply as you did in the simplistic case. Any smidgen of implication as you just did in the simplistic case is erased in total by the ramifications of allowing this nonsense to go on. Especially when there is a perfectly acceptable procedure that people should be following, and which many (most that I’ve had) people DO FOLLOW.

    You people trying to scrimp and save a couple grand here and there and relying on b s “implications” that have no support in the lawl should recognize that you’re living dangerously and if the courts slap you down, so be it.

  46. 105

    Ned,

    I assume that you place a lot of credence in case citations. Instead of turning myself in to the PTO disciplinary authorities for some re-education, I suggest that after reading the following that you check your meds.

    Ping, you do realize, do you not, that in the present case, as in Bristol-Meyers, the applicant filed a continuation with all claims and presented all of them for examination

    592 F.3d 1340, 1343-44:

    The ‘812 patent is the third in a chain of related patents, all of which share a common specification.

    While the ‘947 application was pending, the applicants filed U.S. Patent Application No. 07/124,197 (the “‘197 application”) as a divisional of the ‘947 application.

    The respective claims of the ‘197 and ‘947 applications were therefore divided as between applications along the lines of demarcation drawn by the examiner in the restriction requirement.

    The ‘671 application was filed as a division of the second application in the chain–the ‘197 application.

    Now unless you think that the CAFC should join me at the PTO disciplinary authorities for some re-education, I expect a thank you from you for setting you straight.

    Or would that be self-evidently spuriouos?

  47. 104

    My original restriction would say more than that. My restriction states that A is a different invention from B. My restriction states that A is a different invention from C. My restriction states that B is a different invention from C.

    Yes, of course your restriction requirement says all those things.

    Now, suppose I elect A without traverse and present B and C in a single divisional. Suppose you choose not to restrict between B and C. We’ll take that as an implicit statement by the PTO that B and C aren’t so different after all.

    That failure to restrict “un-says” that B is a different invention from C. But it says nothing about the relationship of either invention with A. Your last word on the subject clearly states that each and every claim in my parent application is a different invention from each and every claim in my divisional. My parent and my divisional are clearly demarcated along a boundary that you set in your restriction requirement. That should be enough consonance that you can’t reject B and C over A for double patenting.

  48. 103

    “Too bad, you’re wrong.”

    IANAE is correct. You have no idea what consonance means.

  49. 102

    “Whether the applicant is estopped from traversing, I couldn’t say.”

    Why can’t you say? Under what theory would applicant be estopped from traversing?

  50. 101

    “6, I would say that a divisional claiming both B and C is consonant with a restriction requirement between A, B, C.”

    Too bad, you’re wrong.

    “Your restriction requirement states that A is a different invention from B and C, and the divisional doesn’t cross the line demarcating between A and any other invention. ”

    Now we see why you’re wrong.

    My original restriction would say more than that. My restriction states that A is a different invention from B. My restriction states that A is a different invention from C. My restriction states that B is a different invention from C.

    Otherwise my restriction was improperly written and/or issued. We are required to detail the relationship between all groups to each other group. You ever wonder why some of your restrictions (all the proper ones where there are a lot of groups) have a sht ton of paragraphs detailing the relationship between each group with each and every one of the other groups individually? This is why.

    I assume in the instant case that the examiner did at least do this properly. If he didn’t but they didn’t traverse, then they still likely should be screwed as it is presumed that he is saying that they are independent and distinct, even if he didn’t detail it out, otherwise he wouldn’t be restricting from amongst them.

    Now, considering what a proper restriction that I would write says and that we presume all examiners do in valid restrictions, consider your hypo IANAE.

  51. 100

    6, I would say that a divisional claiming both B and C is consonant with a restriction requirement between A, B, C. Your restriction requirement states that A is a different invention from B and C, and the divisional doesn’t cross the line demarcating between A and any other invention. If B and C are presented together in a divisional, I think the most you can do is to again require restriction. Whether the applicant is estopped from traversing, I couldn’t say. I don’t believe you have valid recourse to a double-patenting rejection on these facts.

    If you choose not to restrict between B and C, that’s your decision. You are free to assume the “burden” and examine both “inventions” B and C together. But none of that brings B and C any closer to A for the purpose of double patenting.

  52. 99

    “Not really an answer.”

    Ok, let me be more specific. I wouldn’t assume that myself because there is no reason to. But, if a court did it out of generosity in a case where a simple mistake was likely because people are st pid then I wouldn’t be so much against it.

    “No longer considered to be distinct by who? ”

    By the new examiner.

    “If the examiner found that A and B and C are all independent or distinct from one another, and presented reasons for the independence or distinctness, that’s the end of the inquiry. ”

    I agree. And yes I do understand it.

  53. 98

    “Not really an answer.”

    Ok, let me be more specific. I wouldn’t assume that myself because there is no reason to. But, if a court did it out of generosity in a case where a simple mistake was likely because people are st pid then I wouldn’t fight it.

    “No longer considered to be distinct by who? ”

    By the new examiner.

    “If the examiner found that A and B and C are all independent or distinct from one another, and presented reasons for the independence or distinctness, that’s the end of the inquiry. ”

    I agree. And yes I do understand it. But I’m also saying that I think there is a good chance that a judge would give them a break in an instance where incompe tence was likely the cause of the issue. And I wouldn’t fight it. I wouldn’t personally fight it because I don’t think the restriction law is a good one at all. Be that as it may, I can still read the law and understand how others apply it.

  54. 97

    “If you reject B and C under ODP using the parent application, and you are appealed (petitioned?), you will lose.”

    Maybe. Perhaps the best course of action is to hold the claims non-responsive. I can’t say that I know what the correct course of action for an examiner to take here is, I have not fully explored it myself. But, in either event, that presentation of claims is improper under the lawl that you so adore. Perhaps the office simply shouldn’t give the DIV the filing date of the parent since they failed to make the other invention the subject of a DIV.

    “There is nothing in section 121 that requires that applicant file each group identified in the restriction requirement as a separate divisional.”

    You’re absolutely right that there is nothing explicitly that states that. But then again, there is nothing in 121 that will provide you with a filing date should you fail to do as it says.

    “the other INVENTION (SINGULAR) is made the subject of a divisional application which complies with the requirements of section 120 of this title it shall be entitled to the benefit of the filing date of the original application.”

    The other invention (singular) is taken care of in a DIV one at time under 121 bucko.

    That is to say in effect, the language “the other invention” precludes you from making claims to two independent and distinct inventions as determined in the previous restriction requirement.

    “The applicant believes there is a solid argumnent that there is no serious burden to search and examine all claims to B and C in a single application, but doesn’t have any argument that searching A, B, and C does not present a serious burden.”

    Then that’s a prime example of when the app TRAVERSES IN THE ORIGINAL APPLICATION AS THAT IS WHERE THE RESTRICTION IS BEING MADE. Ya tardbrother.

    “Applicant files ONE divisional application containing B and C. Applicant expects to receive a restriction, at which time Applicant will traverse the requirement on the grounds that there is no serious burden to search and examine B and C in the same application.”

    This presents the exact same problem as I told you guys about wayyyyyyyyy up thread. You in effect nullified the stat period for replying to the restriction requirement in full. Bullsht. If that’s how you want to play it then we should sure as f be holding such responses as you present in A as non-responsive as not being fully responsive. You didn’t answer the rest of the restriction requirement. And if you want to say that your cancellation was sufficient to answer the restriction requirement then we should either not be listening to your bs and still holding you non-responsive or oblige you by rescending the restriction requirement because you’re no longer claiming two or more independent and distint inventions. And we can go around like this (if you choose to add back in the claims after our withdrawal) until you decide to answer the restriction. Or until your patent term is up. You can choose.

    Under standard procedure today we assume that your “without traverse” means that you traverse none of the restriction requirement. Not that you’d like some more time to think about/respond to part of the restriction requirement. And if you did pull the stunt as in your example then you would just about sure as f lose the petition on your traversal in the 2nd app. They might even go ahead and tell you that you should have traversed it in App 1 and that the issue isn’t even ripe for petition any longer. Or they might just cite some consonance caselawl and deny your petition outright.

    And if you’d like to make an issue of that particular nuance I’ll be happy to settle it for us all the next time I get such a bogus reply. Interesting situations like this I’m more than happy to oblige my services on.

    “The patent issuing from the ONE divisional application is not subject to invalidity under ODP from the parent application claiming A. ”

    We’ll see about that. Plainly the MPEP has stated for a long time, and courts acknowledge, that DIV’s without consonance do not get ODP protection.

    See here:

    link to scholar.google.com

    “In addition to the express requirements of section 121, we have also construed the statute to require consonance: the applicant must maintain the line of demarcation between the independent and distinct inventions that prompted the restriction requirement. Gerber Garment Tech. v. Lectra Sys., Inc., 916 F.2d 683, 688 (Fed.Cir.1990). This consonance requirement prevents an applicant from amending [6 note, or simply presenting in the beginning] the claims in the divisional application in a way that would violate the originally imposed restriction requirement and thereby impermissibly extend the patent term as to that subject matter. Id.[4]”

    See here:

    link to scholar.google.com

    “Compliance with a restriction requirement means the claims in a divisional application must be consonant with those not elected under that requirement. Noncompliance with the consonance requirement is normally detected by the PTO examiner. See Manual of Patent Examining Procedure (MPEP) § 804.01 (double patenting protection of Section 121 does not apply where the claims are not consonant with, i.e. “have been changed in material respects 686*686 from”, the claims subject to the restriction requirement). Examiners’ compliance with MPEP § 804.01 may account for the absence of court decisions on the precise fact pattern before us and the consequent “first impression” status of the case in this court.”

    We see there that apparently examiners may have been ODP’ing people for quite awhile in accordance with 804.01.

    “We are presented here also with that procedural quirk in the law whereby the nonappealable becomes reviewable and piecemeal litigation becomes permissible. ”

    “The divisional application must have claims drawn only to the “other invention.” See Lerner v. Ladd, 216 F.Supp. 81, 84, 136 USPQ 624, 626 (D.D.C.1962) (same invention double patenting).”

    That’s from your own lawl.

    “That is not to say, however, that the elected claims may not be looked to in assessing compliance with the prohibition against claiming the same invention in two patents.[2] In this regard, the phrase “obviousness type” may have been an unfortunate choice, for, as here, the claims in the divisional application may be actually drawn (though in variant language) to the same invention as that set forth in the elected claims. As discussed below, that phenomenon may be viewed as a failure to keep the claims in the divisional application “consonant.””

    “Consonance requires that the line of demarcation between the “independent and distinct inventions” that prompted the restriction requirement be maintained. Though the claims may be amended, they must not be so amended as to bring them back over the line imposed in the restriction requirement. Where that line is crossed the prohibition of the third sentence of Section 121 does not apply. Cf. In re Ziegler, 443 F.2d 1211, 1215, 170 USPQ 129, 131-32 (CCPA 1971) (Section 121’s prohibition does not apply where restriction requirement withdrawn because divisional application no longer filed “as a result of” the restriction).”

    That’s your lawl, and while 121 might not go so far as to tell you, the uneducated reader about it, it sure as f does seem to tell some judges about it and some examiners. Gl in your panel when your case comes up.

    Try:

    link to scholar.google.com

    for background, though I feel it is useless in the instant case.

    My final thoughts after having read the cases on consonance and all are that you’re likely very wrong about your ODP rejection scenario.

  55. 96

    “Because people are by and large, rtards and sometimes we can give them a small break. If a court decided not to, I’m fine with that too. In fact I would probably prefer that they didn’t.”

    Not really an answer.

    Using your example, why would you assume that if the applicant filed a divisional application containing both B and C, that “B and C are implicitly no longer considered to be distinct from one another.”

    No longer considered to be distinct by who? The applicant? What difference does that make?
    The examiner makes the determination of whether inventions as claimed are independent or distinct, not the applicant. If the examiner found that A and B and C are all independent or distinct from one another, and presented reasons for the independence or distinctness, that’s the end of the inquiry. The examiner cannot use the claims to A to reject the claims to B or C under ODP, and can’t use the claims to B to reject the claims to A or C under ODP, etc.

    I thought you said you understood restriction.

  56. 95

    “Why would you assume that?”

    Because people are by and large, rtards and sometimes we can give them a small break. If a court decided not to, I’m fine with that too. In fact I would probably prefer that they didn’t.

  57. 94

    “I suspect you are among those attorneys who actually believe that one can file a continuation, call it a divisional and have it be a divisional without satisfying the requirements of the statute.”

    There are no “requirements of the statute” for a case to be a divisional. There are only statutory requirements to obtain the benefit of Section 121 for a divisional. A voluntary divisional is perfectly legal.

    I file a case disclosing a novel eggbeater and a novel computer memory. I claim only the eggbeater. First case is allowed, I file a divisional with new claims to the computer memory. The second case is a voluntary divisional to a disclosed, but not claimed invention.

  58. 93

    “The inventions are A B C. The apps are, for example 1, 2, 3. App 1 has claims to all of A B C. They all GET DIVIDED from one another in app 1. C is divided from B right then. App 2 comes in with B and C, it is improperly presented. Consonance was not maintained with the restriction.”

    Incorrect. There is nothing in section 121 that requires that applicant file each group identified in the restriction requirement as a separate divisional. Like most of what you post, you’re making that up.

    Using your example, the original application is filed with A, B, and C. The examiner issues a restriction requirement. The applicant believes there is a solid argumnent that there is no serious burden to search and examine all claims to B and C in a single application, but doesn’t have any argument that searching A, B, and C does not present a serious burden.

    Applicants elects A without traverse. Cancels B and C.

    Applicant files ONE divisional application containing B and C. Applicant expects to receive a restriction, at which time Applicant will traverse the requirement on the grounds that there is no serious burden to search and examine B and C in the same application.

    No restriction is issued. Instead, a first action allowance is issued allowing all claims to B and C. The patent issuing from the ONE divisional application is not subject to invalidity under ODP from the parent application claiming A.

  59. 92

    “The examiner can repeat the restriction between B and C or he can f the applicant with no ODP protection”

    If you reject B and C under ODP using the parent application, and you are appealed (petitioned?), you will lose.

  60. 91

    “I would be fine with simply assuming that B and C are implicitly no longer considered to be distinct from one another.”

    Why would you assume that?

  61. 90

    ping, I suspect you are among those attorneys who actually believe that one can file a continuation, call it a divisional and have it be a divisional without satisfying the requirements of the statute.

    ping, you need to turn yourself in to the PTO disciplinary authorities for some re-education.

  62. 89

    “Cept you haven’t – you havent told me how C got divided from B if B (sic, app 2?) didn’t have all of B and C to begin with. That was the question.”

    You have a typo in there somewhere tardface. The inventions are A B C. The apps are, for example 1, 2, 3. App 1 has claims to all of A B C. They all GET DIVIDED from one another in app 1. C is divided from B right then. App 2 comes in with B and C, it is improperly presented. Consonance was not maintained with the restriction.

    The examiner can repeat the restriction between B and C or he can f the applicant with no ODP protection, or in this simplistic case, I would be fine with simply assuming that B and C are implicitly no longer considered to be distinct from one another. However, in the later option that can get tricky because if that is assumed to be true then there might not be a basis for having restricted A from B and/or C in app 1 and if you withdraw that restriction as well, well idk, sht hits the fan.

    Assuming the applicant cancelled C above or the examiner repeated the restriction then app 3 comes in with C having been properly restricted from A and B in App 1. If one of the other options was taken at app 2 stage then app is probably a little bit fed.

    That is all a simplistic example though. In the instant case we have the applicant runnin buck wild over the restriction’s groupings all over the dam place.

    “Uh-huh.

    Ya gonna stick with that story?

    Yes, read Gargarsa’s summary of the facts.

  63. 88

    Ned – in the present case, the applicant filed a divisional, not a continuation. The further daughter was also a divisional (self chosen)

  64. 87

    Ping, you do realize, do you not, that in the present case, as in Bristol-Meyers, the applicant filed a continuation with all claims and presented all of them for examination. It was examined without a restriction requirement. Only after receiving a FAOM rejection, did the applicant cancel the claims that he “elected” in the parent. He also cancelled the compound claims he thought would interfere with an Eli Lilly patent and claimed them instead in a still further daughter application.

    Now, the examiner could have, but didn’t, in that FAOM, allow all the claims, including the claims still being prosecuted in the parent.

    Now, ping, follow what I am saying carefully. I will go slow, just for you.

    Assume the applicant paid the issue fee and all claims actually issued in the daughter case, could the PTO enter a double patenting rejection over the issued daughter patent for containing exactly the same claims still pending in the parent?

    I hope you would agree that he could. But that means that the parent had no Section 121 benefit from the daughter, which means that the daughter application was not entitled to Section 121 benefit at all, ever, just as 6 previously contended.

  65. 86

    or one of 6 or 7

    Uh-huh.

    Ya gonna stick with that story?

    already told you tard, C has already been “divided”.

    Cept you haven’t – you havent told me how C got divided from B if B didn’t have all of B and C to begin with. That was the question.

    You said “from A”, but that wasn’t the question sunshine.

    There are principles of law that trancend your slightly different factual situations tard.

    You wouldn’t know a principle of law if it was the only thing in your hands. Care to explain the prinicple that allows you to grab a 121 protection when you use a continuation instead of a divisional? That be some trancendant hand waiving you got there.

    Yous and Ned ought to get your trancend and self-evident spurious purposely avoid the real language of law findings together for a good ‘ol hoot-an-nanny good time.

  66. 85

    “in the case at bar, as in Bristol-Meyers, all claims were filed in the daughter case.”

    If you’re looking for a way to distinguish Bristol-Myers (which I’m not really, but we need a rule consistent with all the caselaw), Bristol-Myers seems to rely somewhat on a restriction in the child being inconsistent with the restriction made in the parent.

    Seems like a divisional is only as good as the most recent restriction requirement in the family, and it doesn’t matter which claims are later presented together without restriction.

  67. 84

    “If the second app issues with both B & C, am I safe?

    Can I ask the Examiner to give me a restriction requirement if he doesn’t do it immediately?

    I and the dissent say maybe maybe not. In the simplistic example you present it would appear that the restriction requirement is simply lifted. However, on the other hand you have not maintained consonance with the restriction requirement and I don’t recommend you do it as you just did as you very well might not be in the clear. I personally have different attorneys that do things both ways. Some file in serial/parallel each restricted group/species. On the other hand I just got someone who included all leftover claims in a DIV where they were previously restricted into many groups. I repeated the same restriction from before (save the previously elected and prosecuted invention) and thus he won’t have any issues. In the instant case however they kept on filing on different groupings of their own making. Clearly not a situation where the restriction was simply lifted implicitly and clearly not a situation where they got a different restriction requirement to justify their filings.

    And yes to your last question. Unless of course he immediately allows the case before you ask.

    “Neither case on point here”

    There are principles of law that trancend your slightly different factual situations tard.

    “6, yous better go back and check the facts of the immediate case agains the exact steps you list here homeboy”

    Hardly. In the instant case they simply threw in everything in the second app and ended up with a few claims left (that happened to not be all directed to one of the examiners groups iirc). Be clear, the examiner made a total of 10 groups, not 3. He then made some wack instructions on what they could elect. The applicant flaunted the restriction requirement’s groupings and election options in the second app as well as in the third and simply filed on whatever groups comprising whichever claims he felt like.

    If, in the case at bar he’d ended up with claims to only one group in the second app after having “realized his mistake” then I’d be fine with that app. If, in the case at bar, he’d ended up with claims to only one group in the third app after having “realized a further mistake” then I’d be fine with it. But the bottom line is he just decided to file and acquire DIVs on whatever groupings of claims he made up himself without paying any regard to the restriction itself.

    Let’s be clear, here’s the facts at play here.

    Examiner says: you have GROUPS 1-10 in this app. Pick one of the following pairings 1,8/1,9/1,10/2,8/2,9/2,10/3,8/3,9/3,10/4,8/4,9/4,10/5,8/5,9/5,10 or one of 6 or 7. Applicant did not traverse (like a fool) or his petition didn’t work out so this asinine restriction is what we’re working with.

    Applicant picked, for example, the 1,8 groups to have examined in the first app.

    He then files for 2-7, 9 and 10 in the second app but ends up with only claims to 9 and 10. (note that 9 and 10 were not grouped together in a grouping in the original groupings or in the election options of the restriction).

    He then files for 2-5 in a third app and it issues that way. (note that 2, 3, 4, 5 are not grouped together in a grouping in the original groupings or in the election options of the restriction requirement).

    “Besides, you aint done yet tell me how C can come from B without B having all that stuff finally in C. Tells me 6 – how do you rightly divide out somethin if that something aint there to be divided out?”

    I already told you tard, C has already been “divided”. In app 1. I’m not sure how you’re having trouble understanding this.

  68. 83

    The short story Ned – ya can’t ask for 121 protection from a restriction requirement that ya don’t divide from. That’s all your case stands for. And that’s not the matter here.

    It’s not that you are at a loss to understand, it’s that your position is at a loss. Whether you understand that or not really don’t matta (except to keep ya from posting the same wrong crap again and again, embarassing yourself).

  69. 82

    as in Bristol-Meyers

    Ned, since you don’t read the Rules that I post, why should I expect you to read teh case law that I post?

    Well, I’ll post anyways. From your errant case, I more than graciously provide the following cites from 1345-48:

    The applicants did not file a divisional application in response to either of the restriction requirements, but instead appealed the final rejection of the claims to the PTO Board of Appeals. In 1977, while that appeal was pending, the applicants filed a continuation application, Serial No. 778,955 (“the ‘955 application”), and abandoned the ‘989 application.

    …Although the 1973 restriction requirement was issued against the ‘989 application, and not against the ‘955 application,…

    …argues that the 1973 restriction requirement was not in effect at the time of the filing of the divisional application that matured into the ‘927 patent…

    …We agree with Pharmachemie. The ‘955 continuation application , which was filed in 1977, began a new proceeding in which all of the original claims of the ‘989 application were once again presented for examination

    So ya see Ned, the fact that a continuation and not a divisional was the key linking point makes your little case cite not only distinguished, but non-precedent. Oh, you can go on fooling the simple-minded, but for anyone who can actually read, ya just barking up the wrong tree.

  70. 81

    “If the second app issues with both B & C, am I safe?”

    I don’t see why not. Both were restricted from A, and they can’t OTDP each other in the same application.

    You also shouldn’t have a problem with filing B as a divisional of A, and C as a divisional of either A or B. Plus you save a restriction requirement and wasted claim fees, and you get the added benefit that the examiner doesn’t think you’re illiterate, since he did explain in writing that A, B, C are all separate inventions.

  71. 80

    I’m still confused. If you want to file one app after another (in serial), how do you do it correctly?

    Original has A, B, C and gets 3-way restriction. What do you file next? I would have always filed B & C in a second application. Is that not correct?

    If the second app issues with both B & C, am I safe?

    Can I ask the Examiner to give me a restriction requirement if he doesn’t do it immediately?

  72. 79

    ping, in the case at bar, as in Bristol-Meyers, all claims were filed in the daughter case. Now you place major emphasis that Bristol-Meyers precedent does not apply here because there all claims were filed in the daughter case, but, since the same thing happened in the present case, I am at a loss to understand your point.

  73. 78

    wow different browsers show the open tags differently.

    this thread shows broke on firefox, but ok on explorer.

  74. 77

    priority date and 121 protections are two separate things sonny child.

    Except bright boy, I wasn’t talking no priority date issue – I was talking about the app being active. Pay attention now and keep up.

    following accepted procedure
    Yes, we all would like the Office to follow accepted procedures now, wouldn’t we. But since they don’t, we gotta make lAW (catch those caps?) that protects the applicants for Office mistakes. That would be 121 in the present.

    where the applicant realized their mistake in presenting B and C subject matter claims in the second app and then cancelling the claims to C in the second app then filing another separate DIV to take care of C.” and That is not what happened in the instant case at all.

    6, yous better go back and check the facts of the immediate case agains the exact steps you list here homeboy.

    Besides, you aint done yet tell me how C can come from B without B having all that stuff finally in C. Tells me 6 – how do you rightly divide out somethin if that something aint there to be divided out? You be getting messed up and not answerin the right questions now.

    Call it realizin their mistake (cause someone was going to call an interfere) – and which the majority says don’t matta (and the minority says squat), and yous got the same fact pattern.

    precedent flaunting

    I lol’s you – you without a clue using legal terms. Or are you listening to ‘ol Ned and his IMHO-Ned Law precedence? If you checked it out for yourself, you’d see that the “critical” link in Bristol was a continuation and not a divisional (hear that trumpet) and that in Zeigler the original restriction was changed and the applicant was given the chance to change the original app. Neither case on point here, and neither precedential to counter the rule that the majority rightly lay the smackdown on the Office with. Jus’ because ‘ol Ned can throw out a case cite don’t mean that he is citing real law.

  75. 76

    “Sooo, you are saying that you can file a divisional referencing an application that is no longer active (A), but can file magically from the child (B) because B is active, even though B does not have an examiner restriction? That’s some creative interpretation.”

    Yes. And btw, priority date and 121 protections are two separate things sonny child.

    “funny guy – somehow you want to apply the restriction requirement of A in either or both of B and C. I say this because I am sure (that since you have mastered the facts) that the examiner(s) in B and C made no restriction, yet your short answer is “file C as a divisional from B”. That’s some serious handwaiving.”

    I don’t see anything handwaiving about following accepted procedure. Yes, I most definitely would like to apply the restriction requirement that was issued. That’s why we issue them, so that they are followed and applied.

    “- just like the majority said, then.”

    That quote you just referenced was not regarding the majority’s position it was regarding a different hypothetical. Specifically where the applicant realized their mistake in presenting B and C subject matter claims in the second app and then cancelling the claims to C in the second app then filing another separate DIV to take care of C.

    That is not what happened in the instant case at all. Had they have followed the examiner’s restriction then they wouldn’t be in the situation they’re in having to go to the CAFC to cry for a precedent flaunting decision.

  76. 75

    It has already been filed with all the materials in A. Where the restriction happened.

    Sooo, you are saying that you can file a divisional referencing an application that is no longer active (A), but can file magically from the child (B) because B is active, even though B does not have an examiner restriction? That’s some creative interpretation.

    The case at hand never (in either B or C to my knowledge) presented claims in accordance with the examiner’s restriction requirement.
    funny guy – somehow you want to apply the restriction requirement of A in either or both of B and C. I say this because I am sure (that since you have mastered the facts) that the examiner(s) in B and C made no restriction, yet your short answer is “file C as a divisional from B”. That’s some serious handwaiving.

    Yes that would appear to have been properly done.” – just like the majority said, then.

  77. 74

    “Case in point – see the majority’s decision.”

    The case at hand never (in either B or C to my knowledge) presented claims in accordance with the examiner’s restriction requirement.

    “I think you should still be entitled to the Section 121 benefit between all three applications.”

    Yes that would appear to have been properly done.

  78. 73

    “Please explain the great Office trick of filing C as a divisional from B when B is not filed with all material (app body and claims) for both B and C.”

    It has already been filed with all the materials in A. Where the restriction happened.

  79. 72

    “The only way it could be maintained is if the examiner thereafter actually restricted B and C.”

    That is correct.

  80. 71

    Ned: “6, if I understand you correctly, merely claiming the B – C inventions in a single daughter application is enough, without more, for the applicant to lose Section 121 benefit as between B and C. The only way it could be maintained is if the examiner thereafter actually restricted B and C.”

    I don’t know that the boundary is exactly there, but I think it’s pretty close.

    To contrive the facts a little more, suppose you file B&C in a single divisional, then realize your mistake and cancel C before a first action is mailed. You later file C in a divisional of the B application, after A has issued. I think you should still be entitled to the Section 121 benefit between all three applications.

    To actually answer your question, then, I would say that you maintain the claim to safe harbor until the PTO does something to suggest to you that it’s okay to have those two claim sets in the same application after all. Until that point, you are completely justified in relying on the earlier restriction requirement to split them up.

  81. 70

    6 chuckles,

    Please explain the great Office trick of filing C as a divisional from B when B is not filed with all material (app body and claims) for both B and C.

    Answer: You cannot. You cannot divide out what is not there.

  82. 69

    No, you could file B then C during B’s pendency and be ok.

    Case in point – see the majority’s decision.

  83. 67

    6, if I understand you correctly, merely claiming the B – C inventions in a single daughter application is enough, without more, for the applicant to lose Section 121 benefit as between B and C. The only way it could be maintained is if the examiner thereafter actually restricted B and C.

    Is this right?

  84. 66

    “then MUST I file separate divisionals to B and C before A issues to benefit from the safe harbor provision against A for both B and C? ”

    No, you could file B then C during B’s pendency and be ok.

    “Or might it be possible, to file a single Divisional to B and C, and see what the Examiner does?”

    That is precisely part of the issue here. I feel like it shouldn’t be allowed (by way of getting an ODP rejection on one or both of B and C claims). And so does the dissent.

    See:

    “If an examiner’s initial restriction requires a patentee to elect only “one of the inventions” to prosecute, see 35 U.S.C. § 121, I see no logical reason why the patentee should be permitted to combine the remaining inventions into a subsequent single divisional application [without suffering the consequence of an ODP rejection].”

    “If so, and the Examiner does not restrict B and C, then does the B&C patent benefit from safe harbor against the A patent?”

    This decision doesn’t specifically touch on that issue, but I would urge no, because consonance with the first restriction req was not maintained. And if the examiner misses making the ODP rejection then tough sht at the DC.

    “On the other hand, if the Examiner restricts B and C, and I elect B, do either B or C benefit from safe harbor against A? If so, which ones: B, C, or both B and C?

    B. C will as well if you file another DIV for it after the restriction.

  85. 65

    NAL, despite its complicated fact pattern, the Feds did say in Bristol-Meyers, and necessarily so, that there was no implied restriction in daughter cases to the extent the examiner did something there inconsistent with the restriction in the parent.

    Implied restriction. That is the question.

    In the case at bar, examination of all inventions in the daughter case is inconsistent with an implied restriction. It is tantamount to a withdrawal of the restriction.

    I think the PTO needs to be able to examine inventions efficiently in the manner it so choses. If it chooses to actually examine the B – C inventions in the daughter case, that is enough, IMHO. The applicant then loses the right to call any subsequent continuation applications a division of the daughter case.

    This matter is not over. This was not an en banc decision. On its face, it is wrongly decided. It is also inconsistent with binding precedent.

    Applicants cannot rely on it at all, but they might. That is why the Feds should have taken it en banc.

  86. 64

    I just want the short answer, and I’m having a little trouble gleaning it from the article and comments.

    Basically what I want to know is, if I get a restriction to A, B, and C, and elect A, then MUST I file separate divisionals to B and C before A issues to benefit from the safe harbor provision against A for both B and C?

    Or might it be possible, to file a single Divisional to B and C, and see what the Examiner does?

    If so, and the Examiner does not restrict B and C, then does the B&C patent benefit from safe harbor against the A patent?

    On the other hand, if the Examiner restricts B and C, and I elect B, do either B or C benefit from safe harbor against A? If so, which ones: B, C, or both B and C?

  87. 63

    As the man with the answers, IANAE, your answer here is confusing.

    To my mind, a multiple-way restriction sets the stage for a later divisional directed to any one of B, C, D, etc. from any co-pending family member.

    conflicts with

    The problem here was that…applicant chose to split them into two applications even though nobody required him to do so

    It seems that you are saying that your mind is the problem, and that the requirement that set the stage was not a requirement so there was no stage.

    And I do not feel any better calling anything an implicit traversal because traversals must be explicit, and the action was not a traversal, but a following of the restriction with respect to B and C.

  88. 62

    “So, what are some practical solutions?
    Invention A, B, C. Restriction required. You select A.
    Then what? If you want to get B & C in the future, how do you do it?”

    I would expect you could file B as a divisional of A and C as a later divisional of either A or B, and you’d probably be fine. To my mind, a multiple-way restriction sets the stage for a later divisional directed to any one of B, C, D, etc. from any co-pending family member.

    The problem here was not the timing of the filings and allowances. The problem here was that an applicant had a pending application (that it happened to itself be a divisional is immaterial) with two sets of claims being examined on the merits, and the applicant chose to split them into two applications even though nobody required him to do so.

    If it makes you feel better, call it an implicit traverse of the restriction with respect to B and C.

  89. 61

    NAL,
    And I thought this whole thing started with an employee of the Office,silly me.
    THEY”LL BE NO MORE TRICKS ON THIS KID.

  90. 60

    “Can anybody explain how that can happen in a divisional?”

    Maybe they were trying to figure out whether the 121 shield applied to the claims that were added that didn’t belong to the restricted invention and whether or not they should make an ODP rejection of the claims they didn’t restrict out? Maybe the PTO legal dept was having a hard time helping them out.

    I think that most likely.

    While I did get a chuckle out of this reply I really do think that most likely. Save perhaps the explanation that the examiner really didn’t want to work on the app because of the manner which the app amended. When apps do this kind of non-standard thing it really f’s with the examiner if they’re a thoughtful examiner NAL. This might come as a bit of a surprise to you.

    “This leads to a unclear way of proceeding for the applicant in order to meet the consonance of any additional applications that may be born after the first restriction is pursued.”

    After having reread the decision I have to agree with you here. Nevertheless, the applicant couldn’t be bothered to get the restriction sorted out in the first instance and was stuck with the ridiculous first restriction req.

    I will agree with you NAL that the original Restriction was a piece of sht. App should never be electing more that one GROUP. A GROUP should stand on its own at all times as they comprise various groupings of claims not restrictable/not restricted from one another. However, that was a petitionable matter, and the app apparently did not avail himself of such recourse or was denied it.

    This:

    “(1) one of the compound groups and one of the method of use groups”

    should never be an option. It is likely that the examiner should have kept the compounds and the methods of use together in one large group, or several smaller groups that each had a compound and method of using that specific compound.

    “However, since the Office CHOSE to start the ball rolling, and the applicant met all of the 121 requirements in A, that defense is now available for all offspring of that decision, whether the offspring is made by conscious decision of the applicant or later in a second divisional action, by the necessity of the Office (I think the potentiality of a second restriction scenario muddies the water quite a bit). The safe harbor exists for all offspring and all relations of those offspring. ”

    That is plainly a bad interpretation of the statute as it exists. I have little qualm with you wanting it to mean that or be amended to say such a thing, but it plainly is nowhere near what is written as of today.

    DIVs have a special purpose NAL, so do CONs. And that purpose is laid down quite clearly for practicioners. Use them for their purpose and not for the purpose of the other. That’s my advice on this whole “mess”. If you and others who feel as you do choose to not take it don’t be surprised when a court tells you that you did things wrong and you lose your patent as a result.

    At least your latest post indicates that you might be competent enough to do this properly in your own cases. I find it infinitely laughable that you continually defend the blatant incompetence of the applicant, while slamming the office for doing exactly what 90%+ of practicioners would have liked the office to do at the time of examination of C: examine the whole app and not give them an ODP rejection on some claims utilizing the grandparent. In 20/20 hindsight of course, it appears blatant that they’d want the restriction. Funny enough, there was no suggested restriction presented to the examiner in app C so far as the facts have been retold. Had such a request been made, I have little doubt the examiner would have obliged. These are exactly the types of things that lawlyers are useful for getting done. This one happened to fail to do it. Boo hoo.

    If this had been a pro se mistake then I might reconsider giving the guy a break. But come on, these are the rules of the game.

    Other than that, I’ve made my peace with this issue, anyone else that wants a say can have the last word.

  91. 59

    Ned,

    I think our unease may be different, but different sides of the same coin. I tend to view the “safe harbor” question and the “but for” prong as the easy prong to satisfy. To me, the whole mess starts with the Office decision to make a restriction requirement (before 6 gets in an uproar, I am not saying that that first decision was a mistake). However, since the Office CHOSE to start the ball rolling, and the applicant met all of the 121 requirements in A, that defense is now available for all offspring of that decision, whether the offspring is made by conscious decision of the applicant or later in a second divisional action, by the necessity of the Office (I think the potentiality of a second restriction scenario muddies the water quite a bit). The safe harbor exists for all offspring and all relations of those offspring. It is only in this manner that I can buy into the decision fixing the B-C mess as acceptable.

    Where I have expressed my unease is in the consonance prong. I see the shortcut that the majority took and I understand the logic employed, but I find that logic very unsatisfying. The facts as I can read them (I do not have the file jacket) do not help me make a case for the Office. The allowance given by the first examiner in his restriction seems to be too indirect – any number of combinations would satisfy the first examiner. This leads to a unclear way of proceeding for the applicant in order to meet the consonance of any additional applications that may be born after the first restriction is pursued. To me, this is an Office-made problem for which the applicant should not be punished. The fact that the Office, with full knowledge of the proceedings decided to address the B application in such different terms is also a fault of the Office.

    6 should realize that in any given application, the examiner is free to request restriction or not – that is not the point here. The point is that this was not just any application. This was an application born as a divisional where the Office already CHOSE to restrict. In other words, this was an application already operating under the veil of 121 protection. Why this was missed by both the examiner and his supervisor boggles the mind. It also boggles the mind that this application lingered for more than four years. Can anybody explain how that can happen in a divisional?

    Back to the consonance issue. While I understand the majority position (and unfortunately see the glaring weaknesses in the minority report), the case law for consonance seems to demand more. I think “blame” can and should go to both the applicant (who appears to have tried to maintain as much of his rights as possible) and to the Office for its handling of the B divisional (with knowledge of the A-121 possibility). Had the Office proceeded properly, the applicant would have had to maintain consonance per the A examiner requirement, and the whole mess would have been avoided. This “but for” reinforces the starting “but for”. Since 121 was in play, and that law expressly protects the interests of the applicant, the only one left to hold the bag is the Office.

    I do not like that the applicant proceeded with “B” and “C” claims mixed, and that the eventual B and C patents do not hold FULL consonance with the A examiner’s restriction, but given that that restriction was not clear and definite as to how exactly to proceed, the onus – by law – belongs to the Office. 121 was rightfully called into play and provides the type of protection to the type of person that it sets out to do. Gajarsa’s attempts fail, in part because he attacks the wrong things and in part, because covering for the Office when such apparent mistakes are evident would be a more grave disservice to the law than what he says is the current result.

    Regarding the lack of a restriction requirement in the B application to restrict “B” and “C”: I see how you might think that this would have the same effect as a conscious decision to withdraw the (not) implied restriction between “B” and “C” (not – because A opened the door – that door was not shut during B’s prosecution). Equating that effect would be an error. Since the restriction is in play from A, and B enjoys the 121 shield, I believe that an AFFIRMATIVE (and not implied) and purposeful statement that the restriction has been lifted would have been proper. I do not have case law for this, but In re Zeigler has a different fact pattern than the present case and is not directly applicable.

    We cannot forget that even though B was being examined by a different examiner, B enjoyed the 121 benefit. No amount of examiner ignorance removes this from B. I think it a fundamental error to allow erasure of an applicant’s protection from Office error (121) to be erased BY YET ANOTHER Office error. That would sanction two wrongs making a right. In this immediate case, the B divisional status directly from A is critical. It appears to be yet another Office mistake in not realizing this.

    Please do not reference Bristol-Meyers. That case does not support anything here because that case’s fact pattern is immensely more intricate.

    You then plead for the en banc so that “damage” would not be done. I ask you to take a moment and view the result from the majority standpoint. From that standpoint, what is the damage that would be avoided with an en banc decision? I ask in this manner, because I think that the majority would distinguish In re Ziegler.

    Curious,

    You are correct in that now-a-days, this would not be so much an issue. The problem here is two-fold. The first is that the terminal disclaimer was disallowed since the preceding item had expired. The second, and (gulp) Ping was correct – is that the real elephant in the room is that this patent too would be expired, BUT FOR the Office delay of over four years.

    Over four years on an application that had been searched on and examined so, so many years previously – an application that had already invoked the 121 protection.

    Protectors of the Office – please explain this.

  92. 58

    I thought you could file applications in serial. First A, then B, and then C. Since they all end at the same time (claiming A as prioirty), you aren’t trying to unfairly extend.

    Wouldn’t filing a terminal disclaimer in C eliminate the problem of obviousness type double patenting?

    Slightly different question: Can A ever be used against C as prior art if you filed the chain (A-B-C) and always claimed priority (continuation or divisional)?

  93. 57

    “So, what are some practical solutions?

    Invention A, B, C. Restriction required. You select A.

    Then what? If you want to get B & C in the future, how do you do it?”

    Uhh.. you file divisionals directed to B and C before A issues.

  94. 56

    Can someone explain Dennis’ title, “Avoid Terminal Disclaimers”? If you find yourself in a questionable situation, isn’t filing a TD the prudent and harmless thing to do?

    (I see “curious” directly above me had the same sentiment).

  95. 55

    Would filing terminal disclaimers make this all moot?

    What is the harm in filing terminal disclaimers these days?

  96. 54

    So, what are some practical solutions?

    Invention A, B, C. Restriction required. You select A.

    Then what? If you want to get B & C in the future, how do you do it?

  97. 53

    Noise, the voluntary nature of the cancellation of the “C” subject matter from the “B” and “C” application is what bothers me. Now I fully understand and I agree that any non-elected subject matter presented in any continuation application, however complicated the prosecution history, has the benefit of section 121 with regard to the claims that were elected in the original application in which the restriction was made. My problem is this: in a subsequent application where there is no restriction requirement made between two inventions, but rather a rejection on the merits entered after an examination, how is it that the voluntary cancellation of the claims directed to one of them be deemed an election that would entitle the voluntarily cancelled claims the benefit of section 121 vis-à-vis the claims remaining after the voluntary cancellation? B and C may have a safe harbor versus A, but C does not have a safe harbor vs. B.

    My hypos are all straight forward. No complications. I assume inventions A B and C. They may have a common point of novelty or relationship, e.g., A is a compound, B is a process of making the compound and C is a use of the compound.

    In the original case, the examiner issues a three-way restriction. The applicant elects invention A, and files a divisional directed to the other two inventions.

    Now I can see how under the majority view that the B and C inventions have the protection of section 121 from a double patenting rejection over a patent that issues on invention A; this regardless of the fact that the latter two inventions were combined in one application rather than being made in separate applications.

    But now let us assume that the examiner in the divisional case decides to examine the two inventions together. He in fact conducts a search and issues an office action, rejecting the claims. By this action, he has essentially stated his belief that the inventions in this case are not in fact independent and distinct and has effectively withdrawn any implied restriction between B and C entered by the examiner in the parent case.

    Now under In re Ziegler, a CCPA case that had cannot be overruled by the Federal Circuit except en banc, a withdrawal of a restriction requirement along with the opportunity to present claims in the case where the restriction requirement was withdrawn, removes Section 121 benefit from the non elected claims, regardless of whether they were already filed in divisionals. When an action on the merits is given against two inventions previously restricted in a parent case, any implied restriction between them is withdrawn. Any subsequent cancellations of these claim are not and cannot be “withdrawals.” Any continuations cannot have the benefit of Section 121 under in re Ziegler vis-a-vis the others claims that were examined.

    It therefor seems to me that the voluntary cancellation of claims after rejection on the merits cannot be, under In re Ziegler or even under Bristol-Myers, considered an act done in response to a restriction requirement. For this reason, the subsequent presentation of the voluntarily cancelled claims in a continuation is not the equivalent to the filing of a divisional application. These claims therefore cannot have the safe harbor benefit of section 121 with respect to the claims remaining in the parent even if they may have section 121 benefit with respect to the claims elected and remaining in the grandparent.

    The panel decision in this case is not good law as it flies in the face of In re Ziegler and Bristol-Meyers. This case should have been taken en banc because of the damage it may do.

  98. 52

    Hey smartie,
    you know who you are. Now I get why nothing is happening. It is because I never had the correct smartie listed.
    SIGNED,
    Dudley Do Right now

  99. 51

    Ned,

    Re: “His cancellation of the “C” claims from the “B” application was completely voluntary”

    This was not assumed otherwise. As the majority pointed out, it is merely immaterial whether the applicant voluntarily did so or not, as the fact pattern still traces back to the A action.

  100. 50

    Ned,

    I am not assuming any facts not in evidence. As you say (and 6 does not understand) in the divisional, all claims were filed.

    I do not say that the “C” claims were restricted out. I do say that they were removed prior to allowance. Perhaps I should have said prior to issuance.

    Your hypos still do not seem clear to me. In your second hypo, are you not claiming “C” in Division B?

    6,

    You and sista Ping make quite a pair. Perhaps you should start your own blog together.

    A few notes:
    “The office never restricted them out” – But they did – see A
    “They could have been presented in the original.” – they were , see A and see B
    “office choosing not to restrict is not an error.” – perhaps correct, but not according to Gajarsa. You would think the world ended if such happened with how tight he wants 121.
    “the office is called upon to act” – Applicant invoked 121 – that’s a call to act (Office had ALREADY started this path with A – that eliminates the “may”.
    “some apples into your oranges” – true, but I’m commenting on Gajarsa’s argument – not a strong one for him do to mix.
    “substantial room for mischief” – agreed – that’s what is so bothersome about the consonance position.
    “waiting until a 3rd app to present claims” – you mean the same claims presented iin both A and B?
    “ODP grandparent app” – Isn’t the issue B-C, not A-C?

  101. 49

    6, your reading of the last sentence of Section 121 is indubitably correct. Good job throughout on this.

  102. 47

    Noise and Ping, you are assuming facts not in evidence. In the divisional, all claims were filed. A rejection was then entered, not a restriction requirement. The applicant was not required to elect any claims. His cancellation of the “C” claims from the “B” application was completely voluntary. They could have issued in the B case had they not been cancelled.

    So please simply answer the question. Had the B and C claims both been allowed, would the subsequently filed C “divisional” from B actually be entitled to Section 121 benefit.

  103. 46

    “OFFICE RESPONSIBILITY and PURPOSE of 121
    The law says, “”The validity of a patent shall not be questioned for failure of the Director to require the application to be restricted to one invention.” The Director failed to follow through when the applicant went after all the other “stuff” after A was divided out. ”

    The director didn’t “fail” to restrict, he chose not to. Those are two separate things lil missy. That sentence plainly is NOT an attempt to give 121 double patenting protection to EVERYTHING IN EVERY APPLICATION EVER IN EXISTENCE JUST BECAUSE THE DIRECTOR CHOSES NOT TO RESTRICT. And it plainly is not an attempt to countermand the MAY they just put in the first sentence. You plainly cannot get 121 DP protection for anything other than an office restricted out invention. It is only a sentence stating that there should not be any rejections based solely upon the notion that it may be grounds for invalidation if there are two or more independent and distinct invention claimed in an application that the director did not restrict.

    Note NAL, that questioning the validity of some claims in a patent based on a ODP grandparent app is entirely different than questioning the validity of a patent for failure of the director to restrict.

    Riddle me this, and riddle me that, but Noise, be so kind as to tell me why these particular applicants didn’t just file a CON with these claims when the grandparent was in the works. Oh, what’s that? They voluntarily chose not to? For no other reason than re tardedness?

    If there could be one thing that the office did make a mistake on in this app it was not making a ODP rejection of those claims based on the grandparent app.

    I’m outta hea.

  104. 45

    Noise do be a darling and state for us, in one sentence or two, what, specifically was the office’s error in this case? I’m a little confused as to what your position is specifically.

    I note that I expect to be laughing when you state that the office SHOULD HAVE ISSUED A RESTRICTION REQUIREMENT IN app C or some nonsense like that. I also note that is no mistake on the part of the office. It is dam near crystal clear that is a mistake on the part of the applicant for waiting until a 3rd app to present claims to a different invention. They need to take some responsibility for their actions.

  105. 44

    Again I note Noise, just as the dissent did, there is substantial room for mischief if you start going your direction.

  106. 43

    “On top of not actually forbidding (and instead being a path of rectifying), the Office WILL itself ask for restriction on something already restricted, thus showing that an applicant may have an application that claims more than one separate and distinct invention – even on an invention already “singularized” by the Office. Sounding all tough and ominous about the severity of 121 (misapplied as a tool against applicants) when multiplicity may sneak by a first or even second pass sounds like a loud clanging wagon (empty). Who hasn’t been chagrined from a late process request for restriction on an item already previously restricted?”

    You seem to be bringing in some apples into your oranges here, but from what I can tell you seem to believe that the fact that the office goes out of its way to be nice about restriction practice somehow effects what protections the law grants applicants. Hmmm, I think not sister. Perhaps I will start being hard arse about this particular thing and we’ll see if we can get some caselaw straightening things out a bit.

  107. 42

    “but the particular application here calls for THE OFFICE to act in a certain way, ”

    How in the sam fin hel do you think the office is called upon to act in some certain specific fashion by a statute that plainly states that they may act or they may not? Do tell.

  108. 41

    “And besides, the 121 shield started with the first examiner, ”

    Also pingerdoodle, you don’t get the 121 shield for inventions that you didn’t bother to present claims to in the first app in order to get yourself a restriction done upon them (or in the species case the examiner didn’t do a species restriction on your non-claimed stuff).

  109. 40

    “genesis (and the “but for”) IS the Office restriction requirement way back in A.”

    Hardly. There is no “but for” in this case’s facts with regards to the claims that are causing the issue. The office never restricted them out. They could have been presented in the original. Now the app just up and decides to present them in a DIV that is for a completely separate invention that WAS restricted off.

    You need to review tha facts girlfriend.

    And, once again pingerdoddle, the office choosing not to restrict is not an error. Ya tard.

  110. 38

    Ned,

    I do not understand what point you are trying to make. It appears that all you have done is play with semantics between the two hypos. For example, in your second hypo you have Division C off of B, which is only possible if the application being divided claims B and C (as in your first hypo).

    Ping,

    Get help and stop bothering people.

  111. 37

    Ned,

    Facts aint your thang, are they?

    But the facts in THIS case are like the previous note. Examiner allows B and C. Applicant voluntarily cancels claims to C and then files another application…

    Sorry Ned, the facts are more than a tad diff.

    In this case, applicant tried for “B” and “C” in B (quotes since B and C reps the app, not the claims) and “C” was not allowed. “C” was removed prior to allowance. And they was removed for the 121 (course, their may have been other reason, but thats no matta).

    Facts: While B is alive, C is done a new app being a divisional off B. Since the gate of 121 was opened from B to A, and “B” and “C” both implicated in A, C earns the 121.

    Facts: majority rules.

  112. 35

    The return of NAL?

    Great timing – the throne room needed more, um, reading material.

    6,

    Why do you say it “should have been”?
    Because them’s the facts here.

    Might help if you checked a few of ’em out before flying with your pants down.

    I agree with you that “It’s 100% completely up to the examiner if he wants to or not in the event that it COULD have been.”, but no one has said that’s what is on here. It aint a COULD have been – it was a WAS (and if you are feeling presidential, the facts of it is “is”).

    The question is whether what “DID have been” should have been allowed. It was up to Mylan to make the case and they couldn’t.

    Twice.

    Your wrongness continues when you claim that “ then wanted to in a DIV later after the split had already been made, since the original filed B did have all the things in C, but was chopped back. A better argument, and one I would have gone with, would have been estoppel.

    The applicant had ample opportunity to add claims to those other species/groups in the parent. AMPLE.
    Not sure what case you be goin on about, but say Hi to sarah for me.

    You aint askin the right questions and you aint got the right facts.

    Good ‘ol Ned’s closer, but still a tad off from the reality

    Here, the first divisional clearly was filed in response to the restriction requirement. thus killing 6’s starting point. Unfortunately for Ned, as the majority point out, it is immaterial how the second divisional came about, because it was dealing with the initial examiner’s points. It don’t matta whether the second examiner was the one requesting a restriction, because the second divisional was actually what was gotten.

    Thems the facts.

    You can’t say “if the second divisional had not been filed“, because yous don’t get to rewrite history. And besides, the 121 shield started with the first examiner, not the second. If, at the time of filing the second application, the status of divisional was not appropriate, somebody at the Office shoulda said something (interlude: man with large green face: Somebody, Stop me!).

    They didn’t. 121 is in.

    Once again Ned, your habit of ignoring small facts like that may seem to make your position stronger, but really only gives me more material to have fun with ya. Mighty sportin of ya!

  113. 34

    Distinguish the fact situation in the previous note with this:

    Restriction A, B and C.

    Division: B.

    Division C off of B.

    I would agree here that C has 121 benefit vs. B.

    But the facts in THIS case are like the previous note.

  114. 33

    Noise, assume restriction between A, B and C.

    Applicant files a divisional claiming B and C.

    Examiner allows B and C.

    Applicant voluntarily cancels claims to C and then files another application directed to C. B issues. C issues.

    Does C have 121 benefit vs. B?

  115. 30

    The moral of the story is that the Office should pay for its screw-ups, not the applicant.

    HOW CHAINED
    Given:
    A – 4731374, App XXX
    B – 4843086, App YYY
    C – 4886812, App ZZZ

    Near as I can tell – and I might be a bit off – using Malcolm’s structure of listing lineage, the Boehringer series should be:

    For B, B divisional of A, App XXX (later amended to Patent 4731374).

    For C, C divisional of B, App YYY (later amended if appropriate to patent), which is divisional of A, Patent 4731374.

    The chain can be done this way because the applicant did file B with all the information necessary to completely support any divisional, B was still active, and C was a proper child (timing wise). To claim that because it was the “applicant’s volition” of giving the status of C as a division and thus not “stemming from” the restriction requirement in A is a smokescreen because choice or not, the designation was couched in the terms that the initial examiner did open. As much as the applicant “decided” to make C a divisional, the genesis (and the “but for”) IS the Office restriction requirement way back in A. That the applicant claims so is enough. And the individual elements of B and C are the leftovers from A.

    AS A RESULT OF
    Unfortunately for the dissent, the majority has a better argument about “as a result of”.

    The majority’s position that “but for” the examiner’s requirement, any divisional material would have been left in the original is unassailable. Simply ask yourself, would any invalidity for double patenting been at issue had not the first examiner started the protection of 121 by requiring restriction?

    Sure, the applicant made things messy, but the original requirement of what was to be divided out for the application that generated the protection of 121 was met (no one is saying that A and B or that A and C fail double-patenting). The screw-up is the B-C line, but the B-C line would not exist, “but for” the Office and the requirement in A.

    CONSONANCE
    On the face of it, the restriction requirement was clear: you have a bunch of “stuff” here, you can choose “stuff” in a number of ways, but you cannot choose everything. I think the examiner, in allowing for a mix/match of choice opened a can of worms.

    Yet I am very uncomfortable about the consonance thing. I am not sold on the majority view for this factor.

    From Pfizer: “This consonance requirement prevents an applicant from amending the claims in the divisional application in a way that would violate the originally imposed restriction requirement“.

    The majority take on “consonance” seems limited to the highlights above in view of “what is in A” versus “what is not in A” (and NOT if the rest of the “stuff” remains separate from each other)
    – did the applicant meet the originally imposed restriction requirement for the originally processed application (A)?
    The answer is yes (for A).
    – did the applicant, then claim anything in the divisional(s) that was in A, and for which double patenting would apply?
    The answer is no (for A).
    I can see THAT as the clear line of demarcation that the majority claims as having been maintained.

    Still, the consonance is not satisfying. I would think that consonance must involve more than one application. The word BEGS to be applied across applications, and the B application violated that original restriction. So did C. But, unfortunately, only in relation to each other and not to A. Even though the Office is free to restrict additional applications, this is unsettling.

    OFFICE RESPONSIBILITY and PURPOSE of 121
    The law says, ““The validity of a patent shall not be questioned for failure of the Director to require the application to be restricted to one invention.” The Director failed to follow through when the applicant went after all the other “stuff” after A was divided out.

    The Office COULD HAVE repeated the same restriction requirement in both B and C (but they didn’t – yet another screw-up), but (a) line of demarcation was maintained for the A and non-A applications (applicant did meet the original examiner’s requirement of separation and pursuit, in A, of what that examiner wanted. The Director’s failure to follow through can be thought of as the fruit of the poisoned tree in a way. The Office poisoned the A tree, and then “screwed the pooch” by messing up the B-C examination.

    The problem is that this B-C Office screw-up ties into the very protection that 121 was drafted for – mistakes of the Office while requiring restriction that impinge on an applicant’s rights to a patent. That opened door from A is damming.

    In answer to 6 and Ned, you are both assuming the basis for your position. The question is not whether the restriction was automatically carried forward or not. To say that it was not, here, is to assume the point that you need to make. It is NOT a matter of “if it could have been”, because it is a matter of “IT WAS”. You cannot ignore this.

    BAD ARGUMENTS
    I think the dissent digs itself deeper because it tries to challenging 121’s purpose and multiplicity and its arguments are wrong on both items.

    When Garjarsa proclaims that section 121 FORBIDS an applicant to prosecute an application claiming more than one separate and distinct invention, he is off. It is not that such is FORBIDDEN by 121; rather, it is that under 121, the Office “may require the application to be restricted to one of the inventions“. This is the point that 6 and Ned wish to make, and it actually works against the position they wish to advocate. The general rule IS permissive, but the particular application here calls for THE OFFICE to act in a certain way, and it is the LAW of 121 that is to protect the applicant from the Office.

    Responsibility belongs to the Office.

    Blaming the applicant for the screw up of the Office, never a good thing, wasn’t taken kindly by the majority. Especially since the law’s real purpose is to PROTECT THE APPLICANT from any inequity of the Office (basically Gajarsa’s second paragraph).

    On top of not actually forbidding (and instead being a path of rectifying), the Office WILL itself ask for restriction on something already restricted, thus showing that an applicant may have an application that claims more than one separate and distinct invention – even on an invention already “singularized” by the Office. Sounding all tough and ominous about the severity of 121 (misapplied as a tool against applicants) when multiplicity may sneak by a first or even second pass sounds like a loud clanging wagon (empty). Who hasn’t been chagrined from a late process request for restriction on an item already previously restricted?

    Bristol Meyers is inapposite to this case, giving that its fact pattern is extremely intricate – and not at all similar to the level here. There is no doubt here that B is rightfully a division of A, wherein Bristol Meyers, the intereceding chain was very much in question.

    Gajarsa also trots out Geneva 349 F.3d 1373 for idea that 121 must be strictly applied, but that case reads slightly different given the different fact pattern (there the actual restriction by the office was under debate – not the case here). Granted he is a judge, but such stretching (dare I say mis-citing) case law makes me question the veracity of the rest of the position.

    Gajarsa should have focused solely on the fact that B-C was not consonant. The problem there, is that the Office screwed up again (and again, and again), and using a tool that is meant to protect applicants against Office screw-ups is difficult when you have so many of them. Having weakened his position with other arguments, it is not surprising to see, even with the majority’s troubling consonance position, that Gajarsa was in the minority.

  116. 29

    Well 6, we seem to agree as well. Gajarsa is clearly right and there appears to be a raging conflict between this case and cases such as Bristol-Meyers that held, it seems, exactly the opposite. Restrictions are not automatically carried forward.

    If no restriction is entered in an application, any claims divided out are subject to double-patenting rejections. This is self evident. That is what actually happened in the case at bar, and the lower court properly found that Section 121 did not apply.

  117. 28

    The Bristol_Meyer case, link to openjurist.org

    has a very similar fact pattern as this case. In the course of deciding that the earlier restriction did not apply to later applications, the Fed panel noted,

    “The record thus does not support the inference that any of the various restriction requirements automatically carried forward, in part or in whole, from one application to the next.”

    Here, the first divisional clearly was filed in response to the restriction requirement.

    However, the examiner in the second case did not enter a restriction requirement between the remaining compounds, processes and uses. No election was demanded and had the second divisional not been filed, a patent may have issued on all “non elected” inventions from the first case.

    Now I can fully see a basis for the applicability of section 121 for any patents issuing on the non elected claims (first application, filed in the divisional) regardless of when and how they were filed vis-a-vis the claims elected in the first application, I cannot similarly see any basis for according Section 121 benefit to the third application vis-a-vis the second because ALL the claims filed in the second applicants could have issued in that application.

    Simplifying: Restriction between A, B and C. Applicant files divisional directed to B and C. No restriction is entered here. B and C could have issued in one patent. So, if C is then filed as a divisional from the B, C application, C has 121 benefit from A, but not from B because C could have issued in the B, C application.

  118. 27

    “”nor why that divisional would not promptly be subject to restriction on the same basis as the parent.”

    It should have been.

    Why do you say it “should have been”? It’s 100% completely up to the examiner if he wants to or not in the event that it COULD have been.

    There is no “fault of the office” or “error of the office” in this situation. There is only an applicant who decided not to include claims to some groups/species at the outset and then wanted to in a DIV later after the split had already been made. The applicant had ample opportunity to add claims to those other species/groups in the parent. AMPLE.

    The only problem, or error, here is that some people think that they should be able to claim whatever they want whereever and whenever they want. Section 121 does not say that nor does it give license to do so.

  119. 26

    Malcolm, I think you actually have to claim divisional status for section 121 benefit. I think this is part of the reason in the present case the Feds accorded divisional status to the inventions in the third, “C,” application. The applicant claimed it was a divisional of B, which was a divisional of A.

  120. 25

    As for the changes in EPO practice: it’s a debacle and the EPO will come to terms with that in a couple years. Worst idea ever.

    Posted by: Malcolm Mooney | May 07, 2010 at 07:19 PM

    ,,,,,,,,,,,,

    Malcolm, two questions:

    1. Would you care to enlarge on that?

    2. The UK Rule since 1949 is that parent and all generations of all divisionals have to allowed within 4.5 years of the earliest declared priority date. The EPO has borrowed the concept (sort of) because it works so well in the UK. Is the UKIPO then the worst Patent Office ever?

  121. 24

    nor why that divisional would not promptly be subject to restriction on the same basis as the parent.

    It should have been.

    The screw up belongs to the Office. What would have happened had the screw up not happened? Appliant would have been able to perfect separate applications for the distinct inventions.

    121 is there to protect applicants from teh screw-ups of the Office. Much as I dislike Big Bizness and shady dealings (and the crap about a terminal disclaimer after the parent was dead was very shady), the Office needs to eat this one.

    Now the real focus here should be on the other Office screw-up: and what’s up with the extension?

    1,564 Days?

    On claims that had been before the Office since when? It’s not like the examiner didn’t know about the other family members, either. This type of gross negligence is the real crime here. People need to be held responsible – I don’t mean just the examiner – how can something sit around for more than four and quarter years without someone in management getting mighty upset.

  122. 23

    Judge Gajarsa wants to play it tight and construe the law strictly, yet the following portion of that law is absent from the discussion in its entirety:

    The validity of a patent shall not be questioned for failure of the Director to require the application to be restricted to one invention.

    Without a closer look at the actual fact pattern, at least one line of demarcation was followed in the original choice of claims to pursue. Is not the applicant able to choose whether or not to pursue any additional claim sets after the examiner makes the initial distinction? Did not the applicant follow the examiner’s wishes, at least to the portion that was pursued initially? It would seem that a second step in the analysis has been glossed over. There is definitely more than one instance of section 121 in play here. The omitted section of the rule lends strong credence to the majority view.

    The fine parsing of the statute as to plurality/singularity of divisions seems overboard. Whether the applicant pursues a path of division also seems overboard as well (for reference, consider section 251, wherein in a reissue situation, the applicant may choose multiple divisions). Whether or not the move coincided with a strategic intent, may be interesting, but does not avoid the fact that the office first directed a restriction. The MPEP guidance on divisionals MPEP 201.06 is quite liberal (e.g. divisional application should set forth at least the portion), also lending credence to the majority view (also see the intermingled use of continuation and division at MPEP 201.07). I can see both sides of the “as a result” argument; however, the initial “but for” seems more pertinent. One considers that in the original scenario, would the applicant have decided to make a division but for the Office request? Once (and once is enough), that request from the office is made, any downstream action necessarily leads back to the “but for” office decision. An applicant may disagree with the exact lines of demarcation (and this happens quite regularly when applicants traverse the examiner’s requirement), but as long as the applicant is not creating completely new lines, the divisions flow from and are a result of office prompting.

    The big picture here is that the purpose of this rule is for the benefit of the applicant against office error, and the error is not just of one type (as the omitted section leads to and Garjarsa leads away from). The part of the rule that was omitted indicates that failures of the office should NOT be held against applicants as tightly as Garjarsa and Dyk want.

  123. 22

    fish sticks: as long as there is consonance does it matter what you call the “continuing” applications?

    This is, more or less, my point.

    When I file continuations of related apps, I will say “This is a continuation of C, which is a continuation of B, now issued as blah blah, which is a continuation of D, now issued as blah blah.

    But if I got, say, a ten way restriction in D, and I serially file on each of the inventions, then what is the language to use? A divisional of C, which is a divisional of B, now issued as blah blah, which is a divisional of A, now issued as blah blah? That’s not really an accurate description of what’s happening. Last time I checked, the MPEP doesn’t give guideance on this.

    It should, though.

  124. 21

    Some attorneys on these boards have never had that happen

    Yeah buddy – especially when dealing with that 6 duuude!

  125. 20

    “I pay the issue fee and file a divisional application with the claims directed towards a second sequence of interest. I get another Notice of Allowance…”

    Some attorneys on these boards have never had that happen MM, you should tell em how it’s done.

  126. 19

    But thinking about it,…

    The restriction in A means that B cannot equal C (because that woulda been a case of prolix claims, not doubly restricted claims).

    I do not see a fact pattern here that a court could uphold a invalid finding for double-patenting of C with B.

    Unless I am missing some spurious self-evident detail that was left out (damm I ha_te it when spurious details are needed).

  127. 18

    as long as there is consonance does it matter what you call the “continuing” applications?

  128. 17

    Ned,

    That question begs further input from Malcolm.

    (what exactly did you file at B time?)

    oooh, I feel so scholarly – quoting myself.

  129. 16

    ping, then if it is not a divisional of B, you would have to agree that C could be rejected/held invalid for double-patenting over B.

    That, in fact, was the holding of the district court that was overturned here.

  130. 15

    I pay the issue fee and file a divisional application with the claims directed towards a second sequence of interest. I get another Notice of Allowance…

    …The first case has already issued. But…

    Malcolm,

    I think I need more information from your hypothetical to make any observations that I would hold to.

    Does your second application (the first divisional) have everything that the original parent did? Everything except for the full set of claims? Does the original as filed have any claims that are self-112 fulfilling (WD-wise) that are not captured in the second application (the first divisional)?

    To steal from Ned (and add a few spurious self-evident details, just because I’m that kinda guy).

    Original filing sees a 3-way restriction, A B and C.

    You pursue A immediately. Obtain allowance. Prior to issue, you file for B.
    (here’s where my questions above provide needed info – what exactly did you file at B time?)

    A Issues.

    You want to file on C.

    If C is not fully contained in B – you are quite possibly screwed. The true parent of C is A (not B – sorry Ned Malcolm’s given facts only say that C was restricted from A), and A is no longer able to have children. There is no restriction in B to spawn C (just one of those minor spurious self-evident details).

    However, if C is fully contained in B – you are golden. C would not be called a Divisional of a Divisional (unless you got a new divisional requirement in B itself), but more likely, C would be a straight-up Continuation of B (if fully contained), or a CIP (if not fully contained).

    You might have a possible problem with the CIP C. If the part of C that makes up the part missing from B (hence the CIP) is in your original (now A), you cannot capture the priority (because A cannot have children) and A may kill the wannabe-devil spawn C.

    Ned is off-base trying to say C is a divisional of B given the fact pattern. Just aint so.

  131. 14

    Malcolm, this was Gajarsa’s point 2. The statute requires the application be filed “as a result of the restriction.” Since C is not filed “as a result of the restriction,” it should not be entitled to the benefit of Section 121.

    However, the case not taken en banc clearly holds the opposite.

  132. 13

    Ned: Before reading this case and the dissent, I would have suggested that C was a continuation of both B and A and subject to a double-patent rejection over either one without the Section 121 safe harbor.

    Really? Because that’s fxcking insane.

  133. 12

    Ned Clearly, this case allows serial filing of divisionals, a practice now all but forbidden in the EPO

    Yes, and I think the serial filing of divisionals must continue to be allowed until some notice is provided to the contrary. The lack of clarity in this regard was one of the main reasons that the infamous “proposed rule changes” sxcked so hard.

    As for the changes in EPO practice: it’s a debacle and the EPO will come to terms with that in a couple years. Worst idea ever.

  134. 11

    Mooney,

    Let’s see 3-way restriction, A B and C. You file on B. A issues. Then you file on C.

    Now, according to the case not taken in banc, C is a divisional of both the parent and grandparent.

    Before reading this case and the dissent, I would have suggested that C was a continuation of both B and A and subject to a double-patent rejection over either one without the Section 121 safe harbor. Had C been filed while A was still pending, it would have been entitled to the Section 121 safe harbor.

    Clearly, this case allows serial filing of divisionals, a practice now all but forbidden in the EPO.

  135. 10

    6, you got me. I see your point. Gajarsa appears to be right on both issues.

  136. 9

    “6, I don’t think traversing has anything to do with your right to file all non elected species in single divisional. ”

    You’re right, it doesn’t, because you have no such “right” according to a clear reading of the statute. But I was just saying that, in the event that you really wanted to file the DIV before you would even have a chance to have had your petition worked on then it would make the most sense to allow such a thing. In any other circumstance then it should be prohibited outright by the language of 121.

    “The filing is tantamount to a request for the next examiner to reconsider the issue of independence anew.”

    By the judiciary’s rulings then there should be no opportunity to do such once the determination was properly made in the first case (and not overtuned on petition).

    “Now you might raise a waiver argument, but I do not buy it without more. Perhaps if you cited some case law?”

    Exactly, you in effect are waiving your ability to get those independent inventions done all at once or request that they be done so. Allowing otherwise basically just gives them extra time to decide to traverse and then file a petition in effect rendering the shortened stat period 1mo and the stat period 6mo for restrictions null. I know of no caselaw on this topic, in fact it may very well be (nearly?) impossible for such to ever be generated due to the nature of petitions vs appeals.

  137. 8

    The problem here is the lack of any guidance from the PTO and/or the lack of any system at the PTO to deal with the practical reality of divisional filings.

    Here’s the biotech situation: I file an application drawn to, say, a method wherein the method comprises doing something with a generically described set of sequences (all of which are novel and non-obvious, of course).

    The Examiner comes back and says: pick a sequence, otherwise the searching is unduly burdensome. So I pick a sequence, and I prosecute the claims and I get a notice of allowance.

    Now it’s time to pay the issue fee and file a divisional. I pay the issue fee and file a divisional application with the claims directed towards a second sequence of interest. I get another Notice of Allowance, and here’s where it gets weird.

    The first case has already issued. But when I write my statement of related applications, I describe the application as …. what? A divisional of a divisional? There is no requirement in the rules, that I am aware of, for me to refer to the original restriction requirement in this “divisional of a divisional”. I could file the broad claims as originally filed in the parent and “see what happens.” Is this second application more appropriately characterized as a continuation of a divisional?

    All the PTO needs to do is lay down the rules and I will be happy to comply.

    As for the questions presented by Mylan, both questions are inane and pretty much answer themselves.

  138. 7

    I think the dissent is a bit misguided where it expresses concerns over unjust patent term extension. Such problems in children applications were taken care of with a change to a 20-year patent term from the earliest claimed US priority date.

  139. 6

    6, I don’t think traversing has anything to do with your right to file all non elected species in single divisional. The filing is tantamount to a request for the next examiner to reconsider the issue of independence anew.

    Now you might raise a waiver argument, but I do not buy it without more. Perhaps if you cited some case law?

  140. 5

    IANAE, we agree.

    Gajarsa cites the case where a divisional application was filed and the restriction withdrawn in the parent. The divisional no longer had divisional status and was invalidated on double-patenting grounds.

    Now here, the examiner does not enter a restriction requirement among the non elected species in the divisional, and a continuation is filed from it. This amounts to a self-denominated, voluntary divisional.

    The lower court holds double patenting between the continuation and its parent. The Feds reverse based on a restriction in the grandparent? But the failure to restrict in the parent, divisional case, is tantamount to withdrawal of the restriction in the grandparent.

    Gajarsa is clearly correct. There is a conflict in panel decisions.

  141. 4

    “Second, while I do not have a problem with a divisional that contains all non elected species,”

    I rather do if they did not traverse.

    “If the other invention is made the subject of a divisional application which complies with the requirements of section 120 of this title it shall be entitled to the benefit of the filing date of the original application. ”

    Note that “other invention” is referring to each individual it does appear when you read this in context with the prior sentence.

    link to uspto.gov

  142. 3

    IMO Garjarsa has got it right.

    “I don’t understand what an applicant has to gain from filing four restricted “inventions” in a single divisional, ”

    They hope that the next examiner (even if it is the original examiner) won’t restrict them, and will examine the whole thing, thus saving the filing/prosecution fees. Indeed, I just had a case similar to this come across my desk this past week which evoked such a response. In that case, they responded to a restriction requirement dividing up two species (I even generously granted them many subspecies within each of those two species against my better judgement) by electing one of the species and presenting claims to yet another species as new claims. I was significantly troubled by this practice and considered going ape nuts on that sht, but let it slide this time because I needed a case to work on right then.

    In the above situation, opinion amongst my fellow examiners was split on what to do in such a case. One said that they elected them by original presentation. However, as they never really got an action on the merits for the species they elected I decided that would probably not be proper. Other examiners stated that they would send another restriction. That was probably what would have been proper had I cared to do it.

    I add that had I have bothered with the second restriction they would have paid for their amending shinanigans dearly as I would have not been so generous with the grouping of the subspecies and that app would have gone ~8 different ways.

    I also note that my failure to do a more thorough restriction has left me feeling rather bad about the app as in doing such a search there simply isn’t enough time alotted to track down such disparate species, and any I didn’t find will simply get allowed.

    Finally I note that I google busted one species right off the bat 😉

  143. 2

    I don’t understand what an applicant has to gain from filing four restricted “inventions” in a single divisional, nor why that divisional would not promptly be subject to restriction on the same basis as the parent.

    However, if for whatever reason the PTO decided to let all four inventions coexist in the first divisional, I agree that it’s appropriate to distinguish the case on that basis and say that the second “divisional” was properly a continuation (or whatever) subject to OTDP. The applicant split into two applications what the examiner would have permitted in a single application, and on those facts we don’t normally give the applicant two independent patents.

    That puts me on the dissent side with Ned, right?

  144. 1

    FUBAR. We have two prior inconsistent decisions and the Feds refuse to take the issue en banc. Now we do not know what the law is because, theorectically, a later panel decision cannot overrule an earlier panel decision.

    Second, while I do not have a problem with a divisional that contains all non elected species, I do have a problem according divisional status from an application without a restriction.

Comments are closed.